68
CMA 1 D Domestic Insitutional Environment of Business 128 Questions [1] Source: CMA 0685 1-31 Sunset laws refer to A. Government support for major business firms in a state of decline. B. Protection of employees nearing retirement. C. Regulatory statutes applicable to certain defense contractors. D. A periodic review of and a fixed termination date for government programs. [2] Source: CMA 1286 1-23 Currently, corporations are chartered by A. The federal government. B. State governments. C. The Securities and Exchange Commission. D. The board of directors of the respective corporations. [3] Source: CMA 1286 1-28 Which one of the following statements is a correct characterization of current federal policy toward political action committees (PACs)? A. Corporations may use company funds to organize and administer a PAC. B. Corporations may make corporate contributions to PACs. C. Unions may help organize PACs, but corporations may not. D. Individuals may only make political contributions through PACs. [4] Source: CMA 1286 1-26 The recent legal concept of strict liability in tort A. Requires consumers to prove that producers were negligent. B. Requires the proof of breach of warranty. C. Permits contributory negligence on the part of consumers to be an acceptable defense by manufacturers. D. Tends to support plaintiff claims against manufacturers of products that are judged to be inherently dangerous. [5] Source: CMA 0687 1-13 Antitrust laws A. Set maximum allowable prices for firms in oligopolistic industries. B. Prohibit firms from collecting price information. C. Require firms with high earnings to provide price and quantity data to the Federal Trade Commission. D. Prohibit firms from allocating customers. [6] Source: CMA 0687 1-15 Tie-in sales (e.g., the sale of camera and film together) are legal A. If the tie-in is necessary to assure product quality. B. If the tying product is patented.

CMAPart1D(Domestic Insitutional Environment of Business)

Embed Size (px)

DESCRIPTION

CMAPart1D(Domestic Insitutional Environment of Business)

Citation preview

Page 1: CMAPart1D(Domestic Insitutional Environment of Business)

CMA 1 DDomestic Insitutional

Environment of Business128 Questions

[1] Source: CMA 0685 1-31 Sunset laws refer to

A. Government support for major business firms in a state of decline.

B. Protection of employees nearing retirement.

C. Regulatory statutes applicable to certain defense contractors.

D. A periodic review of and a fixed termination date for government programs.

[2] Source: CMA 1286 1-23 Currently, corporations are chartered by

A. The federal government.

B. State governments.

C. The Securities and Exchange Commission.

D. The board of directors of the respective corporations.

[3] Source: CMA 1286 1-28 Which one of the following statements is a correct characterization of current federal policy toward political action committees (PACs)?

A. Corporations may use company funds to organize and administer a PAC.

B. Corporations may make corporate contributions to PACs.

C. Unions may help organize PACs, but corporations may not.

D. Individuals may only make political contributions through PACs.

[4] Source: CMA 1286 1-26 The recent legal concept of strict liability in tort

A. Requires consumers to prove that producers were negligent.

B. Requires the proof of breach of warranty.

C. Permits contributory negligence on the part of consumers to be an acceptable defense by manufacturers.

D. Tends to support plaintiff claims against manufacturers of products that are judged to be inherently dangerous.

[5] Source: CMA 0687 1-13 Antitrust laws

A. Set maximum allowable prices for firms in oligopolistic industries.

B. Prohibit firms from collecting price information.

C. Require firms with high earnings to provide price and quantity data to the Federal Trade Commission.

D. Prohibit firms from allocating customers.

[6] Source: CMA 0687 1-15 Tie-in sales (e.g., the sale of camera and film together) are legal

A. If the tie-in is necessary to assure product quality.

B. If the tying product is patented.

C. If used to facilitate price discrimination.

D. Under all circumstances.

[7] Source: CMA 0687 1-22 The Federal Communications Commission

A. Allows free access to the radio spectrum.

B. Limits the number of television stations the major television networks may own.

C. Regulates the profits of radio and television stations.

D. Censors program content on television.

[8] Source: CMA 1287 1-14 An example of social regulation of business is

A. Tariffs on imported lumber products.

B. Rules against discrimination in the hiring of employees.

C. Enforcement of patent laws.

D. Antitrust rules agreements to allocate customers.

[9] Source: CMA 1287 1-16 Requirements by licensors and franchisors that their licensees and franchisees buy inputs from a particular supplier

A. Are legal if they are necessary to assure product quality.

B. Are always illegal unless the input is patented.

C. Are legal if used to facilitate price discrimination.

D. Are legal under all circumstances.

Page 2: CMAPart1D(Domestic Insitutional Environment of Business)

[10] Source: CMA 1287 1-17 The intent of antitrust laws is to

A. Establish a range of allowable profit rates for firms in oligopolistic industries.

B. Prohibit firms from engaging in joint ventures with foreign firms.

C. Require firms with high earnings to relinquish any exclusive patent rights which they own.

D. Prohibit agreements that limit individual firm output.

[11] Source: CMA 1287 1-18 Which one of the following statements regarding the application of antitrust rules to professional fees is true?

A. Most professionals, including physicians, lawyers, engineers, and accountants, are exempt from the antitrust laws.

B. Agreements to increase fees are legal if the quality of services provided improves.

C. Fee discrimination in any form is illegal.

D. Agreements to set either minimum or maximum fees are illegal.

[12] Source: CMA 1287 1-19 The antitrust prohibitions against mergers that would tend to lessen competition

A. Are enforced solely by the Commerce Department.

B. Are enforced solely by the Federal Trade Commission.

C. Are enforced by the Federal Trade Commission and the Justice Department.

D. Were repealed by an Act of Congress in 1982.

[13] Source: CMA 1287 1-20 In regulating intrastate local utilities, state public utility commissions

A. Usually approve rate schedules that set the same rates for residential, commercial, and industrial customers.

B. Use only original cost methods of rate base valuation.

C. Permit utilities to refuse service to some customers within their jurisdiction.

D. Usually set rates of return for utilities based on the cost of capital as reflected in the capital markets.

[14] Source: CMA 1287 1-21

If Federal Aviation Administration regulation of airline passenger service increases the variable costs of individual firms in the industry by requiring greater expenditures on quality checks and maintenance, then

A. In the long run, the industry will tend to have more firms producing lower levels of output.

B. Fixed costs will rise.

C. Prices will fall in the long run.

D. Prices will rise in the short and long run.

[15] Source: CMA 1289 1-26 The two major functions of the Federal Trade Commission are

A. Antitrust actions and consumer protection.

B. Antitrust actions and regulation of import quotas.

C. Regulation of railroads and airlines.

D. Consumer protection and monitoring labor union practices.

[16] Source: CMA 1289 1-24 All of the following are characteristic of the Taft-Hartley Act except

A. The prohibition of a closed shop.

B. The guarantee of the workers' right to organize.

C. States are encouraged to pass right-to-work laws.

D. The provision of descriptions of unfair labor practices committed by unions.

[17] Source: CMA 1289 1-29 The basic purpose of the securities laws of the United States is to regulate the issue of investment securities by

A. Providing a regulatory framework in those states that do not have their own securities laws.

B. Requiring disclosure of all relevant facts so that

investors can make informed decisions.

C. Prohibiting the issuance of securities that the Securities and Exchange Commission determines are not of investment grade.

D. Channeling investment funds into uses which are economically most important.

[18] Source: CMA 1289 1-30 Which one of the following statements best describes how regulatory agencies of the U.S. government are restricted in the adoption of specific regulations?

A. Regulations must be consistent with standards established in the congressional act which created the agency.

Page 3: CMAPart1D(Domestic Insitutional Environment of Business)

B. All proposed regulations must be cleared with a central coordinating agency, the General Accounting Office.

C. The agencies must present proposed regulations to all affected parties for comment.

D. Businesses subject to the regulation must be notified 1 year before the regulation will be put into effect.

[19] Source: CMA 0690 1-2 The Consumer Product Safety Commission (CPSC) has been called by its critics "the most powerful regulatory agency in Washington." The most likely reason for this concern is that the CPSC

A. Covers all consumer products.

B. May ban the production and sale of a product until it has formulated a standard.

C. Does not permit an industry group to adopt a voluntary standard.

D. May order the recall of products it has determined to be unsafe.

[20] Source: CMA 0690 1-5 Truth-in-lending is one form of price standardization that, since the adoption of the Consumer Credit Protection Act on July 1, 1969, has been provided by U.S. government regulations. The purpose of this legislation is to

A. Regulate the amount of interest that may be charged.

B. Prohibit the use of appraisal fees.

C. Allow immediate wage garnishment by creditors.

D. Disclose the finance charge and the annual percentage rate.

[21] Source: CMA 0690 1-6 Equal employment opportunity is concerned with, among other issues, the recruitment and selection of employees. A U.S. Supreme Court ruling in 1971 involved the use of pre-employment tests (Griggs v. Duke Power Co.). In that case, which was determined under Title VII of the Civil Rights Act of 1964, the Court ruled that pre-employment tests

A. Given to minorities, regardless of the reason given, violate Title VII.

B. Are legal, as long as there is no intent to discriminate.

C. In any form are illegal.

D. Must be directly related to job requirements.

[22] Source: CMA 0691 1-21 A major policy of the Equal Employment Opportunity

Commission (EEOC) is

A. To restrict enforcement of the Equal Employment Opportunity Act of 1972 to relatively small companies in order to minimize disruptions.

B. To rank the goal of employee productivity ahead of the goal of equal employment opportunity considerations.

C. To avoid legal remedies to achieve the goals of the EEOC.

D. To have businesses achieve employment mixes reflecting the local labor pool of protected groups.

[23] Source: CMA 0691 1-24 State laws that make it illegal to require union membership as a condition for getting or holding a job are known as

A. Fair trade laws.

B. Fair employment laws.

C. Open shop laws.

D. Right-to-work laws.

[24] Source: CMA 0691 1-30 An industry that has not experienced significant deregulation is the

A. Airline industry.

B. Banking industry.

C. Trucking industry.

D. Steel industry.

[25] Source: CMA 1291 1-19 Over the years, various groups and activities have been exempted from coverage by the U.S. antitrust laws. Which one of the following statements regarding such exemptions is correct?

A. All sales and purchases of products by American firms outside of the U.S. are automatically exempt from coverage by the antitrust laws.

B. All resale price maintenance agreements between manufacturers and retailers selling the manufacturers' product brands are exempt from the federal antitrust laws.

C. Under certain conditions, a person who wishes to form an export trading company may obtain a certificate of antitrust immunity from the Commerce Department after concurrence by the Justice Department.

D. Water common carriers in foreign trade may enter into price fixing and market sharing treaties or agreements provided the agreements are ratified by the U.S. Senate before being put into effect.

Page 4: CMAPart1D(Domestic Insitutional Environment of Business)

[26] Source: CMA 1291 1-20 The Environmental Protection Agency (EPA) might control pollution by setting effluent standards for maximum discharge or by constructing a sliding-tax charge based on the amount of effluent emitted. One advantage of the sliding-tax charge is that

A. It will totally eliminate pollution.

B. There is greater business incentive to discover new methods of controlling pollution.

C. It increases the necessity for the EPA to fully understand pollution control technology.

D. It increases the degree of direct government intervention in the economy.

[27] Source: CMA 1291 1-21 With respect to the federal antitrust laws, regulated industries are

A. Completely exempt.

B. Covered to the same extent as private industries.

C. Covered to the extent determined by the applicable regulatory agency.

D. Covered to the extent determined by statute and the courts.

[28] Source: CMA 1291 1-22 Federal regulatory agencies do not have power to

A. Impose agency taxes on private industry.

B. Issue rules and regulations.

C. Investigate violations of statutes and rules.

D. Recommend penalties for violations of statutes and rules.

[29] Source: CMA 1291 1-24 Violations of federal regulations on equal employment opportunity are sometimes inferred from the fact that a firm has a very small percentage of protected groups among its employees. These regulations are sometimes attacked on the grounds that they may be in conflict with the economic concept that

A. The applicant who would be most productive for the job should be hired.

B. The work force employed should be in the same proportion as the demographics of the total population.

C. Equal opportunity strengthens the seniority system.

D. Wages are inversely related to marginal productivity.

[30] Source: CMA 1291 1-25

The general approach to regulation by the Consumer Product Safety Commission is to

A. Levy fines against the producers of unsafe products.

B. Set safety standards for various products.

C. Increase the freedom of consumer choice.

D. Reduce the cost of regulated products.

[31] Source: CMA 1291 1-26 Social regulation, as exemplified by the Occupational Safety and Health Act and by the Environmental Protection Act, is frequently alleged to be inefficient. The perception by firms is that this inefficiency appears to be a result of

A. Concern for the quality of life.

B. Lack of concern for the magnitude of marginal benefits relative to marginal costs.

C. The use of flexible rather than rigid standards by bureaucrats.

D. Low compliance costs to business firms.

[32] Source: CMA 1291 1-28 The National Labor Relations Act, also known as the Wagner Act, guarantees labor certain rights. This Act

A. Requires management to make detailed financial reports to the National Labor Relations Board.

B. Designates a list of unfair labor practices on the part of unions.

C. Guarantees the right of self-organization and the right to collective bargaining with management.

D. Requires that elections of union officers be regularly scheduled and that secret ballots be used.

[33] Source: CMA 1291 1-29 The Taft-Hartley Act attempted to restore balance between the rights and obligations of employees and those of employers by

A. Permitting strikes in cases of health and safety violations.

B. Outlawing the union shop.

C. Designating a list of unfair labor practices on the part of employers.

D. Designating a list of unfair labor practices on the part of unions.

[34] Source: CMA 1291 1-30 The concept of comparable worth has become an important issue in the U.S. in recent years. This concept addresses the comparability of

A. U.S. wage rates with foreign wage rates adjusted

Page 5: CMAPart1D(Domestic Insitutional Environment of Business)

for differences in exchange rates.

B. Wage rates for different jobs requiring different skills.

C. Wage rates for the same job in union and non-union environments.

D. Wage rates for the same job in the government and the private sector.

[35] Source: CMA 0693 1-11 The basic purpose of the Securities Exchange Act of 1934 was to

A. Regulate outstanding securities.

B. Regulate new issues of securities.

C. Protect investors from investment losses.

D. Ensure the public that stock market crashes like that of 1929 would not occur again.

[36] Source: CMA 0693 1-14 The private placement of debt securities has all of the following advantages over offerings to the general public except that a private placement

A. Does not require Securities and Exchange Commission regulation.

B. Can be completed in less time than public offerings.

C. Has no flotation expenses.

D. Can include indentures that are "tailor-made" to the borrower's needs.

[37] Source: CMA 0693 1-13 Blue-sky laws are

A. The detailed regulations of the Securities and Exchange Commission designed to prevent fraudulent or misleading security issues.

B. Regulations of the Securities and Exchange Commission governing the treatment of goodwill in the sale of securities.

C. State laws designed to prevent fraudulent or misleading security issues.

D. Local laws regulating the issuance of securities for companies whose operations affect the environment.

[38] Source: CMA 0693 1-19 Many social regulations exempt small business from their application. This exemption has been enacted because

A. Small businesses have no significant impact on jobs and the environment.

B. There are too many small businesses, and funds provided are inadequate to regulate them effectively.

C. Compliance costs are a burden for small firms, and Congress has chosen to exempt such firms.

D. The lobby for small businesses has been so active that Congress has chosen to exempt such firms.

[39] Source: CMA 0693 1-20 The Clean Air Act of 1970 charged the Environmental Protection Agency (EPA) with all of the following responsibilities except

A. Setting standards without consideration for the cost of compliance to manufacturers.

B. Nullifying the right of an individual citizen to bring suit to enforce standards as such suits will now be initiated by the EPA.

C. Setting standards to prevent damage to items such as visibility, crops, and buildings.

D. Establishing minimum ambient air standards for the entire country.

[40] Source: CMA 0693 1-21 The federal Consumer Product Safety Commission (CPSC) was created in 1972 and administers an act covering those consumer products not already regulated by the federal government. Specifically, Section 15 of the Act (CPSA) requires manufacturers to take corrective steps for a product that fails to comply with a product safety rule or contains a defect that could create a substantial product hazard. This provision is included because

A. The CPSC is dependent on the manufacturers to provide information on product safety.

B. Voluntary standards are preferred to compulsory ones, and this pushes regulation back into the hands of manufacturers.

C. It encourages the CPSC to apply cost-benefit analysis in formulating and implementing consumer product safety standards.

D. The manufacturer may be compelled to publicize this defect to consumers and/or refund the purchase price.

[41] Source: CMA 0693 1-22 In recent years, a number of consumer groups have used a variety of methods to persuade companies to change behavior. Which one of the following weapons has not been used by consumer groups in recent years?

A. Boycotts or refusals to purchase from the offending company.

B. Shareholder resolutions at annual shareholder meetings to direct a company's board of directors to do or not to do certain things.

C. Creation of a competing company to replace the offending company.

Page 6: CMAPart1D(Domestic Insitutional Environment of Business)

D. Cooperation through meetings that attempt to find common ground for action by both parties.

[42] Source: CMA 0693 1-23 Which one of the following is not a characteristic of the Occupational Safety and Health Administration (OSHA)? OSHA

A. Encourages labor-management committees to formulate safety and health programs.

B. Inspections are primarily focused on health issues, including long-term exposure to such substances as asbestos and cotton dust.

C. Has the authority to levy monetary penalties on non-compliant employers.

D. Inspections are primarily reactionary; that is, take place after major injuries have occurred.

[43] Source: CMA 0693 1-24 Regulated industries, particularly those subject to industry-specific economic regulation, have several common characteristics. Which one of the following characteristics is not associated with most regulated industries?

A. These industries are capital intensive.

B. The social side effects or consequences of the actions of these firms may be undesirable.

C. Such industries are often described as "affected with a public interest" or considered vital industries.

D. Most of these industries are permitted to earn a return that is equitable to all consumers.

[44] Source: CMA 0693 1-26 Antitrust suits, particularly civil rather than criminal suits, may be brought by many different individuals or agencies. Which one of the following does not have power to bring such suits.

A. U.S. Department of Justice.

B. A state attorney general, when authorized by a state antitrust law.

C. A private party, such as a competitor.

D. U.S. Department of Commerce.

[45] Source: CMA 0693 1-25 Supreme Court interpretations of the Sherman Act of 1890 have determined that certain types of agreements, conspiracies, or combinations are in and of themselves so restrictive of competition as to be conclusively presumed unreasonable restraints of trade. These "per se" violations do not involve the application of a "rule of reason" but are considered illegal merely because they exist. Which one of the following offenses is the most serious, i.e., which one has produced the most criminal antitrust violations?

A. Price-fixing or tampering with the price structure.

B. Market allocation or division of customers, markets, or production.

C. Concerted refusals to deal or boycotts involving any third party.

D. Tie-in sales or any effort to force a buyer to purchase less desirable products in order to purchase the desired product.

[46] Source: CMA 0693 1-27 The impact of successful prosecution for restraint of trade violations of the Sherman Antitrust Act has been less than many proponents hoped for because

A. Only a limited number of criminal prosecutions have ever been filed under Section 1 of the Act.

B. Most companies avoid behaviors that might be construed as antitrust violations because of the likelihood of fines and even jail sentences.

C. The predominant criterion for criminal prosecution is market share; a company's intent is inconsequential.

D. The courts created in the beginning a "rule of reason" that required determining whether competition was lessened by the company's behavior.

[47] Source: CMA 0693 1-28 The first major federal equal job opportunity law since the 1970s is the Americans with Disabilities Act (ADA) of 1990. This act does not

A. Ban discrimination against people with disabilities in employment.

B. Provide tax incentives for compliance costs.

C. Ban discrimination against people with disabilities in transportation.

D. Provide federal funds for the implementation of ADA by employers and those providing public accommodation.

[48] Source: CMA 0693 1-29 In recent years, governmental bodies have tried a number of devices to encourage environmental stewardship. At least fourteen major federal acts or amendments to acts were passed between 1969 and 1990. Which one of the following is not a governmental device used in recent years?

A. Direct regulation or legally enforceable environmental quality standards.

B. Pollution charges or fees for undesirable waste released by the firm.

C. Relocation of facilities to offshore sites where governmental standards are less stringent.

D. Buying and selling of pollution rights involving companies whose emissions are below the standard.

Page 7: CMAPart1D(Domestic Insitutional Environment of Business)

[49] Source: CMA 0693 1-30 Pharmaceutical companies must be in compliance with regulations set forth by the Food and Drug Administration (FDA). The FDA is responsible for all of the following except

A. Allowing extensions to patent lives of pharmaceutical companies in order to recover time lost during the premarket FDA regulatory review.

B. Considering benefit-risk trade-offs when evaluating the safety of drugs prior to approval.

C. Maintaining purity standards and regulations concerning drug composition and potency.

D. Approving drugs that lack substantial evidence that they will have the effects that are represented in advertisements.

[50] Source: CMA 1293 1-1 The Sherman Antitrust Act

A. Is intended for international corporations.

B. Prohibits labor unions from forming closed shops.

C. Outlaws all monopolies.

D. Discourages firms from monopolizing.

[51] Source: CMA 1293 1-2 In a regulated industry such as electricity or gas,

A. The forces of demand and supply determine price.

B. Firms can enter or exit the industry easily.

C. Firms can decide on the quality of product or service.

D. Regulatory bodies determine the quality and set the price.

[52] Source: CMA 1293 1-4 The primary reason for social regulation (Clean Air Act, Water Pollution Control Act, Food and Drug Act, etc.) is that

A. The free market provides minimal safety and environmental protection.

B. Consumer and environmental groups are politically powerful.

C. Social benefits from such regulations always exceed the costs.

D. Social regulation is more desirable.

[53] Source: CMA 1293 1-5 An efficient way to control pollution in line with the expectations of the Clean Air Act Amendments of 1990 is to

A. Make pollution illegal.

B. Regulate the amount of pollution a firm can create.

C. Promote social awareness about the harm due to pollution.

D. Allow marketable permits for pollution that firms can buy or sell like any other commodity.

[54] Source: CMA 1293 1-6 A horizontal merger is a merger between

A. Two or more firms from different and unrelated markets.

B. Two or more firms at different stages of the production process.

C. A producer and its supplier.

D. Two or more firms in the same market.

[55] Source: CMA 1293 1-7 In unionized and nonunionized sectors of the economy, labor unions generally

A. Result in wage equality in both sectors.

B. Raise wages in the unionized sector.

C. Raise wages at the same rate in both sectors.

D. Lower wages in both sectors.

[56] Source: CMA 1293 1-8 In relation to labor unions, a closed-shop is where

A. Only those belonging to the union can work.

B. Belonging to the union is not necessary.

C. Collective bargaining is not permitted.

D. Workers have the right to work.

[57] Source: CMA 1293 1-9 The most common form of business organization in terms of number of firms in the United States is the

A. Partnership.

B. Limited partnership.

C. Sole proprietorship.

D. Corporation.

[58] Source: CMA 1293 1-11 Environmental policies undertaken in the United States that attempt to weigh the costs and benefits of environmental improvement

A. Will result in the elimination of all pollution.

Page 8: CMAPart1D(Domestic Insitutional Environment of Business)

B. Cannot work because business firms are not willing to cooperate.

C. Grant tax relief to the firms who pollute the least.

D. Would leave an acceptable amount of pollution.

[59] Source: CMA 1293 1-12 Commercial bank deposits are currently insured to $100,000 per account by the

A. Federal Reserve System.

B. Federal Saving and Loan Insurance Corporation.

C. Federal Deposit Insurance Corporation.

D. U.S. Treasury.

[60] Source: Publisher Which of the following statements about the National Environmental Policy Act (NEPA) is most likely to be false?

A. NEPA requires federal agencies to consider certain potential environmental consequences in their decision-making process.

B. NEPA allows the federal government to bring suit against any private person who violates NEPA's provisions.

C. Under NEPA, federal agencies do not have to give environmental considerations priority over other concerns in their decision-making processes.

D. NEPA augments the power of every existing agency with respect to considering the environmental consequences of their proposed actions.

[61] Source: Publisher Which of the following statements is true with respect to the Clean Water Act (CWA)?

A. Despite the CWA's prohibitions, it allows persons to discharge pollutants into waters subject to its jurisdiction as long as navigation thereon will not be permanently obstructed.

B. The CWA subjects all bodies of water located in the United States, whether flowing or not, to its protection.

C. The notion of protecting waters within the jurisdiction of the United States began with the CWA.

D. The CWA seeks to restore and maintain the physical and biological integrity of the waters of the United States.

[62] Source: Publisher Antitrust laws are intended to

A. Establish set profit percentages for firms in regulated industries.

B. Prohibit firms in the same industry from engaging in joint ventures.

C. Ensure a free and competitive market in which consumer demand dictates prices.

D. Limit competition to increase output and lower prices.

[63] Source: Publisher Pricing agreements are allowable in which of the following situations?

A. An agreement to lower prices for consumers.

B. An agreement to fix prices to eliminate unfair competition.

C. An agreement to stabilize prices reasonably and fairly for consumers.

D. Never.

[64] Source: Publisher To be a monopoly, a firm must

A. Be the sole provider of the product.

B. Have a market share of at least 80%.

C. Have the power to control prices or exclude competition.

D. Have entered into contracts, combinations, or conspiracies in restraint of trade.

[65] Source: Publisher Which of the following firms are subject to the requirements of the Foreign Corrupt Practices Act?

A. All corporations that engage in intrastate or interstate commerce.

B. All corporations required to be registered under the Securities Act of 1933.

C. All corporations required to be registered under the Securities Exchange Act of 1934.

D. All corporations incorporated in the U.S.

[66] Source: Publisher A major result of the Foreign Corrupt Practices Act is that corporations are now required to

A. Keep accurate accounting records and maintain an internal control structure.

B. Prepare financial statements in accordance with U.S. and international accounting standards.

C. Produce information to the SEC on foreign commerce and foreign political party information.

Page 9: CMAPart1D(Domestic Insitutional Environment of Business)

D. Adhere to U.S. auditing standards.

[67] Source: Publisher The Foreign Corrupt Practices Act prohibits

A. Bribes to all foreigners.

B. Small bribes to foreign officials that serve as facilitating or grease payments.

C. Bribery only by corporations and their representatives.

D. Bribes to foreign officials to influence official acts.

[68] Source: CIA 0594 IV-67 Which of the following is not considered to be an advantage of organizing a business as a sole proprietorship?

A. Easy and inexpensive to organize.

B. Allows freedom of action for the entrepreneur.

C. Provides strong incentives to manage the business efficiently.

D. Allows the proprietor to carry out all basic management functions.

[69] Source: CIA 0594 IV-66 In which legal form of business organization do the owners of the business enjoy limited liability?

A. Sole proprietorship.

B. Partnership.

C. Corporation.

D. Monopoly.

[70] Source: CIA 0589 IV-47 Which of the following represents a disadvantage of the partnership form of business organization?

A. Unlimited liability.

B. Double taxation.

C. Complexity in formation.

D. Regulatory requirements.

[71] Source: CIA 0593 IV-47 A limited partnership is typically used to limit the

A. Withdrawals of individual partners.

B. Duration of the partnership.

C. Liabilities of some of the partners.

D. Authority of general partners to manage the

business.

[72] Source: CMA 0694 1-16 The Sherman Antitrust Act of 1890 prohibits

A. Price discrimination, tying contracts, anti-competitive mergers, and interlocking directorates.

B. Unfair competition and deceptive business practices.

C. Mergers without prior notification to the Justice Department.

D. Restraint of trade and monopoly.

[73] Source: CMA 0694 1-17 The Taft-Hartley Act of 1947 was an attempt to restore balance between the rights of employers and those of employees by

A. Outlawing the union shop.

B. Imposing mandatory binding arbitration on both employers and unions.

C. Designating certain labor practices on the part of unions as unfair and outlawing the closed shop.

D. Permitting strikes on the part of federal employees.

[74] Source: CMA 0694 1-18 The Robinson-Patman Act of 1936 prohibits price discrimination unless

A. The price is set to meet a competitor's price quote.

B. The price reflects cost increases caused by government specification.

C. The price is set in a market characterized by monopolistic competition.

D. Specific prior approval is obtained from the Justice Department.

[75] Source: CMA 1294 1-10 Which one of the following antitrust laws prohibits price discrimination, tying contracts, anticompetitive mergers, and interlocking directorates?

A. Sherman Antitrust Act.

B. Clayton Antitrust Act.

C. Antitrust Improvements Act.

D. Federal Trade Commission Act.

[76] Source: CMA 1294 1-11 The National Labor Relations Act of 1935, also known as the Wagner Act,

Page 10: CMAPart1D(Domestic Insitutional Environment of Business)

A. Provides for compulsory binding arbitration in the case of national emergency.

B. Guarantees the right of self-organization and the right to collective bargaining with management.

C. Designates a list of unfair labor practices on the part of unions.

D. Requires that union officials be elected on a regular basis and that secret ballots be used.

[77] Source: CMA 1294 1-12 The Labor-Management Relations Act, also known as the Taft-Hartley Act of 1947,

A. Prohibits employers from interfering with the right of employees to form unions.

B. Obligates employers to bargain in good faith with unions.

C. Outlaws yellow-dog contracts, which require employees to remain as nonunion members.

D. Prohibits unions from coercing employees to become union members.

[78] Source: CMA 1294 1-13 The Sherman Antitrust Act

A. Created the Federal Trade Commission.

B. Did not establish penalties for offenders of the Act.

C. Prohibits mergers and acquisitions that substantially lessen competition.

D. Declares illegal, attempts at restraint of trade.

[79] Source: CMA 1294 1-14 Social legislation, as exemplified by the Occupational Safety and Health Act and Environmental Protection Act, is frequently criticized for being inefficient because the agencies

A. Use flexible rather than rigid standards.

B. Impose compliance costs that are low compared to the benefits received by society.

C. Rely heavily on the free market to allocate resources.

D. Rarely consider the marginal benefits relative to the marginal costs.

[80] Source: CMA 1295 1-20 With respect to federal antitrust laws, regulated industries are

A. Completely exempt.

B. Covered as determined by statute and the courts.

C. Covered as determined by the Department of Justice.

D. Covered to the same extent as any other industry.

[81] Source: CMA 1295 1-21 Social regulation is often criticized by industry as inefficient.

Firms perceive this inefficiency to be a result of

A. The failure to consider the marginal benefits relative to the marginal costs.

B. Lenient enforcement policies.

C. Concern for the quality of life but not the quality of products.

D. The use of the internal revenue tax code instead of strict compliance penalties.

[82] Source: CMA 1295 1-22 Which one of the following transactions would be considered a violation of the Robinson-Patman Act?

A. The sale of goods of like quality at different prices to two different wholesalers, both of whom are located outside the United States.

B. The sale of goods of like quality within the United States at different prices based on cost differences related to the method of delivery.

C. The sale of goods of like quality within the United States at different prices to two different wholesalers; all parties are located within the same state.

D. The sale of goods of like quality within the United States but across state lines at different prices to two different wholesalers in the same geographic area.

[83] Source: CMA 1295 1-23 Which one of the following examples of corporate behavior would most clearly represent a violation of the Sherman Act?

A. A retailer offers quantity discounts to large institutional buyers.

B. The members of a labor union meet and agree not to work for a specific firm unless the starting wage is at least $10 per hour.

C. Two firms that are in different, unrelated industries merge.

D. Two firms in the same industry agree in a telephone conversation to submit identical bids on a government contract.

[84] Source: CMA 1295 1-24 The Clayton Act, as amended, prohibits all of the following except

A. Tying contracts that require a customer who is

Page 11: CMAPart1D(Domestic Insitutional Environment of Business)

buying one product to buy a related but perhaps unwanted product.

B. Price discrimination by sellers.

C. Interlocking directorates in large competing organizations.

D. Unfair and deceptive business practices, such as misleading advertising.

[85] Source: CMA 1295 1-25 The acquisition of a retail shoe store by a shoe manufacturer is an example of

A. Vertical integration.

B. A conglomerate.

C. Market extension.

D. Product extension.

[86] Source: CMA 1295 1-26 Blue sky laws are

A. Federal laws that make it unlawful to use deceptive practices in the sale of securities.

B. Federal laws that limit the amount of air pollution in a specific geographic area.

C. State laws that regulate the sale of securities.

D. State laws that regulate the environment.

[87] Source: CMA 1295 1-27 Which one of the following federal acts requires unions to retain financial records and submit financial reports to federal authorities?

A. Taft-Hartley Act of 1947.

B. Wagner Act of 1935.

C. Securities Exchange Act of 1934.

D. Landrum-Griffin Act of 1959.

[88] Source: CMA 1295 1-28 Which one of the following is not exempted from federal antitrust regulation?

A. Labor unions.

B. Intrastate commerce.

C. Patents and copyrights.

D. Telecommunications companies.

[89] Source: CMA 1295 1-29 Listed below are five federal agencies.

I. Food and Drug Administration.

II. National Highway Traffic Safety Administration.III. Consumer Product Safety Commission. IV. Occupational Safety and Health Administration. V. Environmental Protection Agency.Which of the above agencies are considered social agencies?

A. I, II, and IV only.

B. I and II only.

C. I, II, and III only.

D. All of the agencies.

[90] Source: CMA 1295 1-30 The two major functions of the Federal Trade Commission are

A. Antitrust actions and the regulation of foreign trade.

B. Import quality inspections and anti-dumping measures.

C. Antitrust actions and consumer protection.

D. Price discrimination and unfair trade practices.

[91] Source: CMA 0685 1-34 Membership in labor unions as a proportion of the total work force has declined in the past decade. The reason most often given for this decline is that

A. The expansion in employment has occurred primarily in service industries.

B. Unions have been less aggressive in organizing.

C. Workers have become disenchanted with union leaders.

D. A larger proportion of union members have reached retirement age and the replacement workers are not joining the union.

[92] Source: CMA 0685 1-35 Business organizations find that union behavior is changing due in part to the change in composition of union membership. A change that is not occurring in union membership is that on the average there are more

A. Members from minority groups.

B. Women members.

C. Members from blue-collar jobs.

D. Members from the public sector.

[93] Source: Publisher Corporate social responsibility is

A. Effectively enforced through the controls envisioned by classical economics.

Page 12: CMAPart1D(Domestic Insitutional Environment of Business)

B. Defined as the obligation to shareholders to earn a profit.

C. More than the obligation to shareholders to earn a profit.

D. Defined as the obligation to serve long-term, organizational interests.

[94] Source: Publisher A common argument against corporate involvement in socially responsible behavior is that

A. It encourages government intrusion in decision making.

B. As a legal person, a corporation is accountable for its conduct.

C. It creates goodwill.

D. In a competitive market, such behavior incurs costs that place the company at a disadvantage.

[95] Source: CMA 0696 1-18 Resale price maintenance is an example of

A. Horizontal price fixing.

B. Vertical price fixing.

C. Preemptive buying.

D. Tying arrangements.

[96] Source: CMA 0696 1-21 A main provision of the Securities Act of 1933, as amended in 1934, is the requirement that

A. Bonds be issued only under a trust indenture approved by the Securities and Exchange Commission (SEC).

B. Public utility holding companies register with the SEC.

C. New securities offered for sale in interstate commerce be registered with the SEC.

D. All security brokers be licensed by the SEC.

[97] Source: CMA 0696 1-22 The Clayton Act of 1914 prohibits

A. Closed-shop labor unions.

B. Sellers' price discrimination.

C. Group boycotts.

D. Oligopolies.

[98] Source: CMA 0696 1-23 The Federal Trade Commission Act, as amended,

A. Outlaws interlocking directorates in large competing corporations.

B. Prohibits price discrimination by sellers.

C. Forbids tying contracts that require a purchaser to buy an unwanted product to get another.

D. Forbids unfair and deceptive business practices, such as misleading advertising.

[99] Source: CMA 0696 1-24 If a product is believed to pose an imminent hazard to consumers, the Consumer Product Safety Commission (CPSC)

A. May seek a court order to have the product banned or seized.

B. May ban or seize the product upon a majority vote by the commissioners of the CPSC.

C. May ban or seize the product, but only after a notice has been published in the Federal Register for 30 days.

D. Can only ask the manufacturer voluntarily to withdraw or recall the product.

[100] Source: CMA 0696 1-25 Which one of the following questions can be asked by an employer during an employment interview ensuring compliance with anti-discrimination rules?

A. "How old are you?"

B. "Do you have a disability?"

C. "Are you married?"

D. "Do you have references?"

[101] Source: CMA 0696 1-26 A factory moves from a non-right-to-work state to a right-to-work state, and the union continues to represent the employees. In the right-to-work state,

A. The employees can be forced to join the union before they start to work.

B. The employees can be forced to join the union after thirty days on the job.

C. The employees can decide not to be involved with the union.

D. The employees can be forced to make payments to the union.

[102] Source: CMA 0696 1-27 Which one of the following approaches to regulation best describes mandatory labeling information concerning food content and nutritional values?

A. Unregulated markets subject to antitrust

Page 13: CMAPart1D(Domestic Insitutional Environment of Business)

enforcement.

B. Disclosure of all relevant information.

C. Changes in liability rules toward a stricter liability.

D. Creation of marketable property rights.

[103] Source: CMA 0696 1-28 Which one of the following forms of law is created when regulatory agencies transform statutes into regulations and enforcement procedures?

A. Constitutional law.

B. Statutory law.

C. Administrative law.

D. Judicial law.

[104] Source: CMA 1296 1-20 A franchisor's requirement that its franchisees buy inputs from a particular supplier is

A. Illegal according to the provisions of the Robinson-Patman Act.

B. Illegal according to the principles of common law.

C. Legal, but the franchisor must receive approval from the Federal Trade Commission.

D. Legal as long as such a requirement is necessary to assure product quality.

[105] Source: CMA 1296 1-21 A basic purpose of the securities laws in the United States is to regulate the issuance of investment securities by

A. Requiring disclosure of all relevant information so that investors can make informed decisions.

B. Prohibiting the issuance of non-investment grade securities.

C. Ensuring that all shareholders have an equal vote in the election of a board of directors.

D. Providing a regulatory framework for those states that do not have their own securities laws.

[106] Source: CMA 1296 1-22 The Sherman Antitrust Act

A. Established the Federal Trade Commission.

B. Prohibits collective boycotts.

C. Prohibits price discrimination.

D. Established the concept of a patent.

[107] Source: CMA 1296 1-23 The Antitrust Improvements Act of 1976

A. Requires pre-approval from the Federal Trade Commission for interlocking directorates.

B. Prohibits price discrimination.

C. Requires notification prior to a merger.

D. Prohibits restraints of trade and monopoly.

[108] Source: CMA 1296 1-24 Sunset laws refer to

A. Protection of employees nearing retirement.

B. Federal laws that prohibit the sale of securities for companies that have not been in business for at least 3 months.

C. State laws that prohibit the sale of alcohol on Sunday.

D. A periodic review of, and a fixed termination date for, government programs.

[109] Source: CMA 1296 1-25 All of the following are legal rights of shareholders in U.S. publicly traded companies except the right to

A. Vote on major mergers and acquisitions.

B. Receive dividends if declared.

C. Vote on charter and bylaw changes.

D. Vote on major management changes.

[110] Source: CMA 1296 1-29 The Americans with Disabilities Act (ADA) of 1990

A. Permits employers to inquire about a job applicant's prior health insurance claims.

B. Bans discrimination against employees with physical disabilities, but not those with mental disabilities.

C. Requires organizations with 25 or more

employees to provide reasonable accommodation for employees with disabilities.

D. Provides federal funds to employers who implement its provisions.

[111] Source: CMA 1296 1-30 The Insider Trading and Securities Fraud Enforcement Act of 1988 in conjunction with the Insider Trading Sanctions Act of 1984

A. Enacted monetary penalties for insider trading for the first time.

B. Increased monetary penalties for insider trading.

C. Enacted criminal penalties for market manipulation

Page 14: CMAPart1D(Domestic Insitutional Environment of Business)

and securities fraud.

D. Empowered the Securities and Exchange Commission with the authority to recover gains resulting from the illegal use of insider information.

[112] Source: CMA 0697 1-22 A large public company that is well-known can reduce the time required to register and issue securities by using a(n)

A. Shelf registration.

B. Indenture agreement.

C. Secondary market registration.

D. Red herring registration.

[113] Source: CMA 0697 1-25 Which one of the following antitrust laws prohibits price discrimination and other exclusionary practices that may give certain firms a competitive advantage over other firms in the same market?

A. Celler-Kefauver Act.

B. Federal Trade Commission Act.

C. Sherman Act.

D. Robinson-Patman Act.

[114] Source: CMA 0697 1-26 Which one of the following laws addresses the issue of insider trading?

A. Federal Trade Commission Act.

B. Securities Exchange Act.

C. Clayton Act.

D. North American Free Trade Agreement.

[115] Source: CMA 0697 1-27 Airlines headquartered in the United States have experienced deregulation in many aspects of their business. Which one of the following aspects of the business is still subject to regulation by the federal government?

A. On-time departure standards.

B. Domestic fare schedules.

C. Collective bargaining.

D. International fare schedules.

[116] Source: CMA 0697 1-28 All of the following are criticisms generally made of federal regulatory agencies and policies except

A. Taxes imposed by agencies being too high.

B. Political considerations influencing policies and

rules.

C. Regulatory rules being rigid and hard to change.

D. Regulations increasing the cost of products and services.

[117] Source: CMA 0697 1-29 All the following are functions of the Securities and Exchange Commission except the

A. Review of stock trades by corporate insiders.

B. Regulation of interstate offerings of new securities to the public.

C. Setting of rules concerning the proxy process of large public companies.

D. Determination of fair trading prices for the common stock of large public companies.

[118] Source: Publisher The Comprehensive Environmental Response, Compensation, and Liability Act (CERCLA), also generically known as the "Superfund," applies to the release of "hazardous substances." Which of the following is statutorily included in the definition of the term "hazardous substance"?

A. Crude oil.

B. Gasoline.

C. Asbestos.

D. Natural gas.

[119] Source: Publisher Which of the following statements about the National Environmental Policy Act (NEPA) is most likely to be incorrect?

A. NEPA requires federal agencies to consider environmental consequences in their decision-making process.

B. NEPA allows the federal government to bring suit against any private person who violates NEPA's provisions.

C. Under NEPA, federal agencies do not have to give environmental considerations priority over other concerns in their decision-making processes.

D. NEPA augments the power of existing agencies with respect to considering environmental consequences of proposed actions.

[120] Source: Publisher Which of the following statements regarding the Clean Water Act (CWA) is correct?

A. It allows persons to discharge pollutants into waters subject to its jurisdiction as long as navigation thereon will not be permanently obstructed.

Page 15: CMAPart1D(Domestic Insitutional Environment of Business)

B. The CWA subjects all bodies of water located in the United States, whether flowing or not, to its protection.

C. The notion of protecting waters within the jurisdiction of the United States began with the CWA.

D. The CWA seeks to restore and maintain the physical and biological integrity of the waters of the United States.

[121] Source: CMA 1294 1-9 (Refer to Figure 4.) One way of measuring government performance with respect to antitrust regulation is cost-benefit analysis. In the graph, P1 and Q1 represent the price and quantity under a monopoly. Effective government regulation could lower the price to P2 which could increase the quantity to Q2. On the graph, the area ABC represents the

A. Total cost of producing the additional output.

B. Excess unsatisfied demand caused by the regulation.

C. Efficiency gain from antitrust action.

D. Total benefit generated by the change.

[122] Source: Publisher Sheila is a financial manager who has discovered that her company is violating environmental regulations. If her immediate superior is involved, her appropriate action is to

A. Do nothing since she has a duty of loyalty to the organization.

B. Consult the audit committee.

C. Present the matter to the next higher managerial level.

D. Confront her immediate superior.

[123] Source: Publisher Corporate social responsibility is

A. Effectively enforced through the controls envisioned by classical economics.

B. Defined as the obligation to shareholders to earn a profit.

C. More than the obligation to shareholders to earn a profit.

D. Defined as the obligation to serve long-term, organizational interests.

[124] Source: Publisher A common argument against corporate involvement in socially responsible behavior is that

A. It encourages government intrusion in decision

making.

B. As a legal person, a corporation is accountable for its conduct.

C. It creates goodwill.

D. In a competitive market, such behavior incurs costs that place the company at a disadvantage.

[125] Source: Publisher Integrity is an ethical requirement for all financial managers/management accountants. One aspect of integrity requires

A. Performance of professional duties in accordance with applicable laws.

B. Avoidance of conflict of interest.

C. Refraining from improper use of inside information.

D. Maintenance of an appropriate level of professional competence.

[126] Source: Publisher Under the express terms of the IMA Code of Ethics, a financial manager/management accountant may not

A. Advertise.

B. Encroach on the practice of another financial manager/management accountant.

C. Disclose confidential information unless authorized or legally obligated.

D. Accept other employment while serving as a financial manager/management accountant.

[127] Source: CMA 1 According to Statements on Management Accounting Number 1C (SMA 1C) (revised), Standards of Ethical Conduct for Practitioners of Management Accounting and Financial Management, a practitioner has a responsibility to recognize professional limitations. Under which standard of ethical conduct would this responsibility be included?

A. Competency.

B. Confidentiality.

C. Integrity.

D. Objectivity.

[128] Source: CMA 2 At Key Enterprises, the controller is responsible for directing the budgeting process. In this role, the controller has significant influence with executive management as individual department budgets are modified and approved. For the current year, the controller was instrumental in the approval of a particular line manager's budget without modification, even though significant reductions were made to the budgets submitted by other line managers. As a

Page 16: CMAPart1D(Domestic Insitutional Environment of Business)

token of appreciation, the line manager in question has given the controller a gift certificate for a popular local restaurant. In considering whether or not to accept the certificate, the controller should refer to which section of Statements on Management Accounting Number 1C (SMA 1C) (revised), Standards of Ethical Conduct for Practitioners of Management Accounting and Financial Management?

A. Competency.

B. Confidentiality.

C. Integrity.

D. Objectivity.

Page 17: CMAPart1D(Domestic Insitutional Environment of Business)

CMA PART 1 DDomestic Institutional

Environment of BusinessANSWERS

[1] Source: CMA 0685 1-31

Answer (A) is incorrect because it is not meaningful in this context.

Answer (B) is incorrect because it is not meaningful in this context.

Answer (C) is incorrect because it is not meaningful in this context.

Answer (D) is correct. Sunset laws provide for a periodic legislative review of and a fixed termination date for regulatory acts and agency enabling statutes. The legislative body must take affirmative action to determine a program's usefulness and to reauthorize it. The objective is to avoid the creation of programs that continue after their purposes have been achieved.

[2] Source: CMA 1286 1-23

Answer (A) is incorrect because all private corporations are created under state statutes since no federal incorporation statute exists.

Answer (B) is correct. Corporations do not exist at common law. They are entirely creatures of statutory law. In the U.S., private corporations are chartered by the individual state governments.

Answer (C) is incorrect because all private corporations are created under state statutes since no federal incorporation statute exists.

Answer (D) is incorrect because a private corporation can only come into being if the state incorporation statute is complied with. The corporation must exist before directors can be elected.

[3] Source: CMA 1286 1-28

Answer (A) is correct. Federal law prohibits business corporations from donating to political election campaigns at the federal level. Note that this does not prohibit donations to state election campaigns, although some states have such laws. Political action committees (PACs) have therefore grown in importance over the past decade as a means of circumventing this prohibition.

Answer (B) is incorrect because direct corporate contributions are not allowed. However, corporations, trade associations, and unions may form PACs and encourage employees to participate, and company funds may be used to organize and administer the PAC.

Answer (C) is incorrect because direct corporate contributions are not allowed. However, corporations, trade associations, and unions may form PACs and encourage employees to participate, and company funds may be used to organize and administer the PAC.

Answer (D) is incorrect because individuals may give funds either directly to candidates or to PACS.

[4] Source: CMA 1286 1-26

Answer (A) is incorrect because the concept of negligence requires proof of fault.

Answer (B) is incorrect because the theory of strict liability requires neither a warranty nor even privity of contract.

Answer (C) is incorrect because contributory negligence is not a defense in a strict liability case, but comparative negligence may be pleaded in some states.

Answer (D) is correct. Strict liability may be imposed in product liability cases although the defendant was not at fault. In a suit against the manufacturer, a plaintiff must prove that the product was defective, the defect rendered it unreasonably dangerous to a user, the seller was engaged in the business of selling the particular product, and the product reached the user without substantial change from the condition in which it was sold. However, absolute, not strict, liability is imposed for damages caused by inherently dangerous products.

[5] Source: CMA 0687 1-13

Answer (A) is incorrect because antitrust laws do not regulate prices. However, prices may not be discriminatory or collusive.

Answer (B) is incorrect because collecting price information is lawful.

Answer (C) is incorrect because antitrust laws apply to all firms engaged in interstate commerce, regardless of size.

Answer (D) is correct. The Sherman Antitrust Act of 1890 was the first U.S. antitrust law. It prohibited contracts, combinations, and conspiracies in restraint of trade and made monopolizing or attempting to monopolize any part of interstate commerce a criminal offense. Allocating customers among firms within an industry, for example, by division of markets, is a per se violation of the Act.

[6] Source: CMA 0687 1-15

Answer (A) is correct. Tie-in sales involve a seller/lessor's requirement that the buyer/lessee purchase/lease another distinct product in order to obtain the first. They are generally prohibited under the Clayton Act of 1914 unless a small company is attempting to enter a market or the tie-in is needed to protect the firm's goodwill (e.g., by maintaining a

Page 18: CMAPart1D(Domestic Insitutional Environment of Business)

certain quality standard).

Answer (B) is incorrect because tie-in sales requirements are generally illegal if the seller has enough market power to restrict competition.

Answer (C) is incorrect because tie-in sales requirements are generally illegal if the seller has enough market power to restrict competition.

Answer (D) is incorrect because tie-in sales requirements are generally illegal if the seller has enough market power to restrict competition.

[7] Source: CMA 0687 1-22

Answer (A) is incorrect because the FCC allocates access to the radio spectrum to those who will best use the medium.

Answer (B) is correct. The FCC is an independent agency formed in 1934 to regulate all methods of communications over the public airwaves. The functions of the FCC include allocating transmission frequencies, issuing transmission licenses, fostering effective and efficient use of communication resources, and regulating common communications carriers engaged in interstate and foreign operations. Among the rules is a limitation on the number of television stations that can be owned by a television network.

Answer (C) is incorrect because profits are not regulated. Radio and TV station owners may make whatever profits they can as long as the airwaves are used consistently with "the public interest, convenience, and necessity."

Answer (D) is incorrect because program content is not censored (but inappropriate programming can result in future loss of license).

[8] Source: CMA 1287 1-14

Answer (A) is incorrect because tariffs are to protect businesses, not society as a whole.

Answer (B) is correct. A social regulation is one for the good of society as a whole. Its purpose is to reach behavior that is thought to be not purely financial or economic. The prohibition of discrimination in employment and housing are examples.

Answer (C) is incorrect because patent laws are to encourage new product development and protect inventors.

Answer (D) is incorrect because such rules regulate economic activity.

[9] Source: CMA 1287 1-16

Answer (A) is correct. The Clayton Act prohibits exclusive dealing arrangements in which the effect may be to substantially lessen competition or tend to create a monopoly in any line of commerce.

However, this prohibition does not apply if such contracts are designed to assure product quality.

Answer (B) is incorrect because these contracts are not always illegal, and patents have nothing to do with these provisions.

Answer (C) is incorrect because price discrimination is prohibited under the Robinson-Patman Act of 1936.

Answer (D) is incorrect because exclusive dealing is generally prohibited under the Clayton Act.

[10] Source: CMA 1287 1-17

Answer (A) is incorrect because profits and prices are not set by antitrust laws other than to the extent that price discrimination is prohibited.

Answer (B) is incorrect because firms may enter into joint ventures with foreign firms.

Answer (C) is incorrect because patents are available to all inventors, regardless of size.

Answer (D) is correct. Antitrust laws are designed to promote more efficient allocation of resources, greater choice for consumers, greater business opportunities, fairness in economic behavior, and avoidance of concentrated political power resulting from economic power. Competition results in greater output and lower prices than other market structures. Agreements to limit output of a product would lessen competition and increase the price to consumers.

[11] Source: CMA 1287 1-18

Answer (A) is incorrect because professionals and their organizations are not exempt from the antitrust laws.

Answer (B) is incorrect because agreements among competitors to increase fees are illegal per se.

Answer (C) is incorrect because fee discrimination is not illegal per se since many factors influence the fees that a particular professional might charge. For example, a professional might charge a lower price if a service is to be performed during a normally slow period.

Answer (D) is correct. Lawyers, public accountants, and other professionals have received scrutiny from the FTC in recent years concerning pricing policies. In general, professionals cannot make agreements that provide for either minimum or maximum fees since price fixing by collusion among competitors is a per se violation of the Sherman Act.

[12] Source: CMA 1287 1-19

Answer (A) is incorrect because the Commerce Department has no authority to enforce the antitrust laws.

Answer (B) is incorrect because the Justice

Page 19: CMAPart1D(Domestic Insitutional Environment of Business)

Department also has the authority to enforce antitrust laws.

Answer (C) is correct. The FTC, in conjunction with the antitrust division of the Justice Department, has broad authority to enforce the antitrust laws. Since mergers may lessen competition or tend to create a monopoly under the terms of the Clayton Act, they are scrutinized by the FTC and the Justice Department.

Answer (D) is incorrect because prohibitions against mergers have not been repealed.

[13] Source: CMA 1287 1-20

Answer (A) is incorrect because users who receive different volumes of service usually pay different rates.

Answer (B) is incorrect because rates are also based on other than original cost when facilities are acquired by one utility from another utility.

Answer (C) is incorrect because utilities are monopolies and are not permitted to refuse to deal with paying customers.

Answer (D) is correct. Public utilities are natural monopolies subject to governmental price controls. State utility commissions establish utility rates by setting rates so that capital providers earn a reasonable return on their investment (the rate base). The rate base is the amount of assets minus depreciation used in the company's operations.

[14] Source: CMA 1287 1-21

Answer (A) is incorrect because an increase in variable costs would decrease the number of firms in the industry.

Answer (B) is incorrect because fixed costs will be unaffected.

Answer (C) is incorrect because prices will increase in the long run if marginal cost increases.

Answer (D) is correct. An increase in variable costs will cause an increase in prices in both the short and long run since the variable costs will have to be covered in both periods. If only fixed costs were increased, only the long run might be affected.

[15] Source: CMA 1289 1-26

Answer (A) is correct. The Federal Trade Commission Act of 1914 created the Federal Trade Commission (FTC) to help provide day-to-day enforcement of the antitrust laws. In addition to having jurisdiction to enforce all of the antitrust laws, the FTC has broad authority to prevent unfair methods of competition and unfair or deceptive acts or practices. The FTC covers virtually all aspects of commerce. Any form of commercial deception is of concern to the FTC.

Answer (B) is incorrect because the FTC does not regulate import quotas. Its operations are purely domestic.

Answer (C) is incorrect because the Interstate Commerce Commission regulates railroads and the Federal Aviation Administration regulates airlines.

Answer (D) is incorrect because labor unions are monitored by the Department of Labor.

[16] Source: CMA 1289 1-24

Answer (A) is incorrect because it is a provision of the Taft-Hartley Act.

Answer (B) is correct. The Labor-Management Relations Act of 1947, or Taft-Hartley Act, placed restraints on unions that resulted in a more even balance of power between labor and management. The Act provides for mediation of disputes, prohibits closed shops, encourages the individual states to pass right-to-work laws, defines unfair labor practices, and provides for an 80-day cooling-off period. It does not guarantee workers the right to organize. This right is guaranteed by the Wagner Act of 1935.

Answer (C) is incorrect because it is a provision of the Taft-Hartley Act.

Answer (D) is incorrect because it is a provision of the Taft-Hartley Act.

[17] Source: CMA 1289 1-29

Answer (A) is incorrect because federal law applies in all states and supplements state blue-sky (securities) laws.

Answer (B) is correct. The basic purpose of the federal securities laws is to provide disclosure of adequate information so that investors can evaluate investments. This is accomplished through complex registration and reporting requirements concerning the issuance and subsequent trading of securities. However, the federal government does not assess the merits of these securities.

Answer (C) is incorrect because the SEC does not determine the merits of securities; it evaluates whether sufficient information is provided.

Answer (D) is incorrect because the securities laws are not intended to influence the investment of capital in more socially or economically beneficial ways.

[18] Source: CMA 1289 1-30

Answer (A) is correct. An administrative agency is a public board, commission, officer, etc. (other than a judicial or legislative body), with limited power to enforce the law, make rules, and adjudicate disputes involving private rights and duties. It may be independent (the SEC, FTC, FCC) or executive (OMB, cabinet departments). These agencies affect almost all aspects of the nation's social, economic, commercial, and political life. The activity of an

Page 20: CMAPart1D(Domestic Insitutional Environment of Business)

administrative agency is an exception to the separation of powers doctrine because the agency has the power to enforce the law (executive power), make rules and regulations (legislative power), and decide disputes (judicial power). Because an agency is not a formally separate branch of government, it exercises only that power delegated to it, and any act outside the scope of the delegation is unconstitutional.

Answer (B) is incorrect because the GAO is the audit arm of Congress and has no approval authority.

Answer (C) is incorrect because proposed regulations are made public in the Federal Register.

Answer (D) is incorrect because there is no 1-year notification requirement.

[19] Source: CMA 0690 1-2

Answer (A) is incorrect because the Consumer Product Safety Act of 1972 exempts certain products, e.g., aircraft, motor vehicles, food, drugs, and cosmetics.

Answer (B) is incorrect because a ban on a product is appropriate only if specific standards will be ineffective.

Answer (C) is incorrect because industry standards are encouraged.

Answer (D) is correct. The CPSC promotes voluntary safety standards, develops and enforces mandatory standards, prohibits unsafe products if safety standards will not be sufficient, recalls hazardous products, furnishes information to

consumers, and works with state and local governments. Thus, if a product is hazardous and presents an unreasonable risk of injury and safety standards will not adequately protect the public, the CPSC may issue an appropriate order banning the product. If a hazard is deemed to be imminent, it may seek a U.S. District Court's authorization to seize the product or halt its distribution.

[20] Source: CMA 0690 1-5

Answer (A) is incorrect because the Consumer

Credit Protection Act merely requires disclosure. It does not regulate interest rates.

Answer (B) is incorrect because the Act did not prohibit the use of appraisal fees.

Answer (C) is incorrect because the Act had nothing to do with wage garnishment.

Answer (D) is correct. The Truth-in-Lending Act applies to creditors that extend consumer credit to individual debtors (not organizations) in amounts of $25,000 or less. For a closed-end credit transaction, e.g., the typical car loan, the total finance charge, annual percentage interest rate, amount financed, late charges, security interest held by the creditor, the number and amounts of payments, due dates, and the

total amount of payments must be disclosed. Open-end credit transactions, such as those involving credit cards, also have specific, detailed disclosure requirements.

[21] Source: CMA 0690 1-6

Answer (A) is incorrect because appropriate tests can be given to all applicants.

Answer (B) is incorrect because the tests are not legal if they are not directly related to job requirements.

Answer (C) is incorrect because the tests are legal if related to job requirements. Professionally developed tests that meet EEOC guidelines are most likely to survive scrutiny.

Answer (D) is correct. The equal employment opportunity laws are designed to prohibit discrimination in employment. Employers do, however, have the right to use selection procedures in hiring as long as the procedures are not discriminatory. Thus, an employer will have a defense under Title VII of the Civil Rights Act of 1964 if it uses professionally developed pre-employment ability tests that are directly related to job requirements.

[22] Source: CMA 0691 1-21

Answer (A) is incorrect because the EEOC deals mostly with large companies.

Answer (B) is incorrect because the EEOC is not concerned with employee productivity.

Answer (C) is incorrect because the EEOC attempts to conciliate disagreements on matters within its authority, but it has no major policy not to seek legal remedies, such as court orders, damage awards, or loss of government contracts.

Answer (D) is correct. The EEOC has set goals and timetables for employers to bring female and minority work forces up to the appropriate percentages as they relate to the available labor pool. The means of achieving these goals, such as affirmative action programs, are often highly controversial.

[23] Source: CMA 0691 1-24

Answer (A) is incorrect because fair trade laws pertain to the prices charged by retailers.

Answer (B) is incorrect because the term fair employment laws is nonsensical in this context.

Answer (C) is incorrect because the term open shop laws is not customarily used to describe laws that bar conditioning employment upon union membership.

Answer (D) is correct. State right-to-work laws are expressly permitted by the NLRA. They allow individuals to work for a unionized employer without being compelled to join the union or pay dues. Moreover, unions must represent all employees in the

Page 21: CMAPart1D(Domestic Insitutional Environment of Business)

bargaining unit, including these free riders.

[24] Source: CMA 0691 1-30

Answer (A) is incorrect because the airline industry has been deregulated.

Answer (B) is incorrect because the banking industry has been deregulated.

Answer (C) is incorrect because the trucking industry has been deregulated.

Answer (D) is correct. In recent years, many industries that were formerly highly regulated have been deregulated to some extent. Airlines, banks, railroads, and trucking have all been deregulated. The steel industry has never been highly regulated. Thus, there was no opportunity for steel to be deregulated.

[25] Source: CMA 1291 1-19

Answer (A) is incorrect because the Sherman Act makes illegal every restraint of trade in interstate or foreign commerce.

Answer (B) is incorrect because the Sherman Act prohibits resale price maintenance.

Answer (C) is correct. The various exemptions from antitrust laws include intrastate commerce, labor unions, regulated utilities, patents, copyrights, and reasonable noncompetition clauses between buyers and sellers of businesses, partners in a partnership, and purchasers of technology or equipment. Agricultural and fishing organizations, financial institutions, broadcasting, transportation industries, and professional baseball also have varying levels of exemption. Moreover, in 1982, Congress enacted the Export Trading Company Act. The purpose of the ETCA is to encourage export of U.S. produced goods and services. Persons who wish to form an export trading company may obtain from the Commerce Department a certificate of antitrust immunity (called a certificate of review) after concurrence by the Justice Department. This certificate is issued if certain conditions are met. These prohibit unfair export competition, substantial lessening of competition or restraint of trade in the U.S., unreasonable effects on U.S. prices, and sale or resale of the goods or services in the U.S.

Answer (D) is incorrect because the Sherman Act makes illegal every restraint of trade in interstate or foreign commerce.

[26] Source: CMA 1291 1-20

Answer (A) is incorrect because pollution would not be totally eliminated. Some firms might find that a low level of pollution is more cost beneficial than total

elimination.

Answer (B) is correct. The setting of effluent standards has often been criticized as an inefficient method of pollution control. Economists prefer a

sliding tax charge based on the amount of effluent emitted. This method is preferred because, as effluent discharge increases, a company's tax increases, providing a strong incentive for firms to discover new methods of controlling pollution. Rather than dictating technology, the tax allows firms to seek out the technology that is the most cost effective.

Answer (C) is incorrect because the EPA would not be directly involved in technology.

Answer (D) is incorrect because the tax would be an indirect method of economic intervention.

[27] Source: CMA 1291 1-21

Answer (A) is incorrect because all regulated industries are not exempt from antitrust laws.

Answer (B) is incorrect because some regulated industries are exempt or partially exempt from the antitrust laws.

Answer (C) is incorrect because Congress, not the applicable regulatory agency, determines which industries are exempt from specific laws.

Answer (D) is correct. Regulated industries such as public utilities and banks may not be covered by the U.S. antitrust laws. For example, the need for monopoly status and tight regulation by state agencies make energy utilities essentially exempt from antitrust laws. Banks and insurance companies, however, are less tightly regulated and are subject to some provisions of the antitrust laws. Thus, different laws apply to different regulated industries.

[28] Source: CMA 1291 1-22

Answer (A) is correct. A federal regulatory agency may regulate some aspect of all industries or may regulate a specific industry in accordance with power delegated by Congress in the enabling legislation. Agency functions include executive, adjudicatory, and rulemaking activities. Such agencies, however, may not impose taxes.

Answer (B) is incorrect because federal regulatory agencies have the power to issue rules and regulations.

Answer (C) is incorrect because federal regulatory agencies have the power to investigate violations of statutes and rules.

Answer (D) is incorrect because federal regulatory agencies have the power to recommend penalties for violations of statutes and rules.

[29] Source: CMA 1291 1-24

Answer (A) is correct. Title VII of the Civil Rights Act of 1964 prohibits employment discrimination on the basis of race, color, national origin, religion, or sex. Under the Act, a pervasive pattern or practice of discrimination, the employer's adoption of a neutral

Page 22: CMAPart1D(Domestic Insitutional Environment of Business)

rule having an adverse impact on a protected class, and the adoption of a neutral rule that perpetuates past discrimination are violations that often must be proved, at least in part, by statistical evidence. A controversial remedy sometimes adopted in such cases is an affirmative action order, which provides preferences to members of the class that previously suffered from discrimination. Affirmative action preferences apply even though the specific persons benefited are not necessarily those who were victimized by illegal discrimination. Affirmative action programs are sometimes criticized because employment preferences are often viewed as reverse discrimination. Moreover, they may not result in the hiring, retention, and promotion of the most productive workers.

Answer (B) is incorrect because a work force in the same proportion as the demographics of the total population is not in conflict with the equal employment opportunity laws.

Answer (C) is incorrect because even a bona fide seniority system may violate Title VII if it is found to perpetuate past discrimination. An affirmative action remedy would necessarily have the effect of weakening such a system.

Answer (D) is incorrect because wages should be directly related to marginal productivity.

[30] Source: CMA 1291 1-25

Answer (A) is incorrect because the CPSC does not levy fines, although it may bring suit for violations of the Act, and a court may levy fines as a result.

Answer (B) is correct. The Consumer Product Safety Act created the Consumer Product Safety Commission, which promotes voluntary safety standards, develops and enforces mandatory standards, prohibits unsafe products if safety standards will not be sufficient, recalls hazardous products, furnishes information to consumers, and works with state and local governments. Thus, if a product is hazardous and presents an unreasonable risk of injury and safety standards will not adequately protect the public, the CPSC may issue an appropriate order banning the product. If a hazard is deemed to be imminent, it may seek a U.S. District Court's authorization to seize the product or halt its distribution. However, the emphasis of the Commission's work is the prevention of problems through standard setting, not the punishment of wrongdoers.

Answer (C) is incorrect because consumer choice is lessened when the CPSC keeps unsafe products off the market.

Answer (D) is incorrect because the cost of regulation results in higher costs for regulated products.

[31] Source: CMA 1291 1-26

Answer (A) is incorrect because quality of life is the primary objective of these laws.

Answer (B) is correct. Economic regulation usually concerns price and service to the public and is ordinarily industry specific. Social regulation has broader objectives and more pervasive effects. It addresses quality of life issues, such as workplace and product safety, pollution, and fair employment practices, and it applies to most industries. Social regulation has been criticized on the grounds that it is costly, contributes to overregulation, may inhibit innovation, increases inflation and may place a disproportionate burden on small companies, thereby having an anticompetitive effect. Another criticism is that regulators are perceived to have little concern for the relation of marginal benefits and costs. For example, achieving total compliance with the standards for treated wastewater may be twice as costly as attaining 95% of those standards. In other words, the relatively small improvements sometimes sought by regulators may be viewed as excessively costly by those who are regulated.

Answer (C) is incorrect because OSHA and the EPA are criticized for maintaining rigid standards that are mandated regardless of cost.

Answer (D) is incorrect because compliance costs are often quite high.

[32] Source: CMA 1291 1-28

Answer (A) is incorrect because the Act does not require financial reports by management.

Answer (B) is incorrect because the Taft-Hartley Act of 1947 designates a list of unfair labor practices on the part of unions.

Answer (C) is correct. The National Labor Relations Act, or Wagner Act, was designed to control the supply of labor while guaranteeing workers the right to organize, that is, to form, join, and assist labor unions. It also prohibited certain unfair labor practices by management. Moreover, it permits employees to elect a collective bargaining agent by majority vote and provides for the NLRB. The NLRB is a federal agency that hears representation cases and determines whether actions by labor or management constitute unfair labor practices.

Answer (D) is incorrect because the Landrum-Griffin Act of 1959 democratized unions.

[33] Source: CMA 1291 1-29

Answer (A) is incorrect because the Act did not permit strikes for health and safety violations.

Answer (B) is incorrect because the union shop is still allowed in many states; the closed shop was outlawed.

Answer (C) is incorrect because the Act limited the power of unions, not of employers.

Answer (D) is correct. After the NLRA was passed in 1935, unions became very powerful, strikes were crippling, and employees were coerced by labor

Page 23: CMAPart1D(Domestic Insitutional Environment of Business)

bosses. The Taft-Hartley Act placed restraints on unions that resulted in a more even balance of power between unions and employers. Among its many provisions, the 1947 Act prohibited certain unfair labor practices by unions: coercion of employees to join unions, discrimination against nonunion employees except when a valid union shop agreement is in place, refusing to bargain in good faith, secondary strikes, featherbedding (payment by employers for work not performed), and charging new members excessive initiation fees.

[34] Source: CMA 1291 1-30

Answer (A) is incorrect because comparable worth does not address comparability of U.S. and foreign wages.

Answer (B) is correct. The equal employment opportunity laws require employees to receive equal compensation for equal work. But compensation based on comparable worth would require workers to be paid for the value of their contributions to the organization or to society, not on the basis of the value established by market forces that may reflect entrenched sexist attitudes. Thus, workers who perform comparable but not equal work, that is, different jobs needing different skills, would be compensated equally. Comparable worth advocates believe that the application of this principle would remedy the underpayment of persons in occupations historically dominated by women. The most severe application of comparable worth law exists in the Canadian Province of Ontario where it has invoked much criticism because of the high costs relative to the benefits derived. Some Ontario companies have spent many thousands of dollars determining the extent of a comparable w orth problem only to have the result show that one or two female workers were underpaid by a few hundred dollars.

Answer (C) is incorrect because comparable worth does not address wage rates for the same job in union and non-union environments.

Answer (D) is incorrect because comparable worth does not address wage rates for the same job in the government and the private sector.

[35] Source: CMA 0693 1-11

Answer (A) is correct. The 1934 Act was designed to regulate securities after initial issuances. Other purposes of the 1934 Act were to provide adequate information to investors and to prevent insiders from unfairly using their information.

Answer (B) is incorrect because the Securities Act of 1933 was designed to regulate new issues of securities.

Answer (C) is incorrect because the objective is not to protect investors from losses, but to provide investors with adequate information upon which to base investment decisions.

Answer (D) is incorrect because the objective was to regulate securities and provide investors with

adequate information. The Securities Acts of 1933 and 1934 were not a direct result of the stock market crash; instead, it was the bankruptcy of Ivar Kreuger, the greatest swindler the world has ever known, that led to the passage of the securities acts.

[36] Source: CMA 0693 1-14

Answer (A) is incorrect because the purpose of a private placement is to avoid SEC regulation.

Answer (B) is incorrect because, in the absence of federal regulation, private placements typically take less time than public offerings.

Answer (C) is correct. Private placements in unlimited amounts are exempted from the Securities Act of 1933. A private placement may be offered only to knowledgeable and sophisticated investors, may be offered to no more than 35 nonaccredited purchasers, and may not be offered to the general public. However, the securities are restricted, and the issuer must exercise reasonable care in determining that purchasers are not underwriters and are purchasing solely for their own investment purposes. Private placement flotation expenses may be lower than with a public offering, but some flotation costs will still be incurred.

Answer (D) is incorrect because a private placement of securities can be tailor-made to the borrower's needs.

[37] Source: CMA 0693 1-13

Answer (A) is incorrect because blue-sky laws are state, not federal, financial regulatory laws.

Answer (B) is incorrect because blue-sky laws are state laws and have nothing to do with goodwill.

Answer (C) is correct. Blue-sky laws are state laws designed to prevent fraudulent or misleading security issues. The name comes from some of the earliest laws that prohibited "everything under the blue skies that is fraudulent."

Answer (D) is incorrect because blue-sky laws apply to all companies in a given state.

[38] Source: CMA 0693 1-19

Answer (A) is incorrect because small businesses do have a significant impact on jobs and the environment; however, they often cannot absorb the additional costs brought on by regulation.

Answer (B) is incorrect because Congress has elected to exempt small businesses from some social regulation because of the cost burden on such firms.

Answer (C) is correct. Social regulation embraces a variety of government requirements related to matters such as consumer protection, environmental quality, employment discrimination, and workplace safety. Small businesses have sometimes been exempted from social regulations, because Congress has felt

Page 24: CMAPart1D(Domestic Insitutional Environment of Business)

that compliance costs would be an excessive burden for small firms. These costs are relatively fixed and are therefore higher per unit of output for smaller than for larger firms. Compliance costs therefore create a competitive disadvantage for small firms relative to large firms.

Answer (D) is incorrect because the cost burden is a more critical factor than small business lobbyists.

[39] Source: CMA 0693 1-20

Answer (A) is incorrect because the standards were established without consideration for costs of compliance.

Answer (B) is correct. Under the Clean Air Act, individuals can initiate or participate in civil enforcement actions. Individuals may also seek damages for personal injuries suffered as a result of violations.

Answer (C) is incorrect because national ambient air quality standards (NAAQS) have been established to prevent damage to items such as visibility, crops, waterways, and buildings.

Answer (D) is incorrect because national ambient air quality standards (NAAQS) have been established with respect to specified pollutants to protect human health and the public welfare.

[40] Source: CMA 0693 1-21

Answer (A) is incorrect because the CPSC is not dependent upon manufacturers to provide information on product safety. Much information is obtained from consumers.

Answer (B) is incorrect because standards are compulsory.

Answer (C) is incorrect because cost-benefit analysis is not a consideration when products are dangerous.

Answer (D) is correct. A manufacturer, retailer, or distributor must notify the CPSC if it has reason to know that a product poses a "substantial product hazard" because it violates a CPSC safety rule or for other reasons. In these circumstances, the CPSC may order the party to notify those affected, repair or replace the product, or submit its own corrective action plan.

[41] Source: CMA 0693 1-22

Answer (A) is incorrect because boycotts are often used by consumer groups attempting to influence a manufacturer's behavior.

Answer (B) is incorrect because shareholder resolutions are sometimes used in an attempt to influence the boards of large corporations.

Answer (C) is correct. Boycotts of a manufacturer's products are the most common tactic used by consumer groups to induce a company to change its

behavior or the nature of its advertising. Boycotts have sometimes been quite successful. Shareholder resolutions have also been made at stockholder meetings, but such resolutions have rarely been successful. Lobbying for new laws is also an often successful tactic. Occasionally, cooperating with the manufacturer to find common ground has been used. Creating a competing company, however, is not a consumerism tactic -- at least not yet.

Answer (D) is incorrect because cooperative meetings are sometimes used by consumer groups.

[42] Source: CMA 0693 1-23

Answer (A) is incorrect because OSHA encourages labor-management committees to formulate safety and health programs.

Answer (B) is incorrect because inspections are intended to determine whether OSHA standards are being followed. These standards relate to workplace safety and health.

Answer (C) is incorrect because OSHA has the authority to levy civil monetary penalties of up to $1,000 per violation and up to $10,000 for a repeat offense.

Answer (D) is correct. OSHA is empowered to conduct surprise inspections to determine whether standards are being met. However, an employer may demand that the inspector obtain a search warrant. Thus, no precipitating event must occur prior to an inspection.

[43] Source: CMA 0693 1-24

Answer (A) is incorrect because most regulated industries, such as public utilities, airlines, and banks, are very capital intensive.

Answer (B) is incorrect because uncontrolled monopoly power may result in excessive prices, poor quality, and inadequate output.

Answer (C) is incorrect because regulated industries are often considered to be vital. Indeed, some are deemed to be sufficiently vital to justify public ownership or the equivalent (e.g., the postal service).

Answer (D) is correct. Regulated industries are usually those in which natural monopolies exist. Regulation is imposed to prevent the abuses that may flourish in a noncompetitive environment and to ensure that consumers receive adequate service at fair prices. Rates are established to provide a fair and reasonable return to investors, not consumers.

[44] Source: CMA 0693 1-26

Answer (A) is incorrect because the Justice Department and the FTC can enforce antitrust laws.

Answer (B) is incorrect because the a state attorney general and the FTC can enforce antitrust laws.

Page 25: CMAPart1D(Domestic Insitutional Environment of Business)

Answer (C) is incorrect because individual citizens and the FTC can enforce antitrust laws.

Answer (D) is correct. Under the Sherman Act, violations may be civil or criminal. Criminal penalties include fines and imprisonment and enforcement is by the Justice Department. Civil suits may also be brought by private parties, who may recover treble damages. The FTC also has specific authority to enforce the Clayton and Robinson-Patman Acts and implied authority to enforce the Sherman Act. State law may also provide remedies for antitrust violations. The U.S. Department of Commerce is not entitled to bring suit for violation of antitrust laws.

[45] Source: CMA 0693 1-25

Answer (A) is correct. The Sherman Act specifies that some type of arrangements between competitors are to be considered unreasonable without inquiry. These are called per se violations. These include: price fixing (agreeing to any price), division of markets (agreeing where to sell), group boycotts (agreeing not to deal with another), and resale price maintenance (limitations on buyer's resale price). The most critical of these in terms of number of violations prosecuted and dollars involved is price fixing.

Answer (B) is incorrect because allocation of markets, although a per se violation, has rarely been a problem.

Answer (C) is incorrect because boycotts involving third parties are not nearly as common as price fixing prosecutions.

Answer (D) is incorrect because tie-in sales are prohibited by the Clayton Act of 1914.

[46] Source: CMA 0693 1-27

Answer (A) is incorrect because many cases have been filed under the Sherman Act; however, the rule of reason has limited the number of successful prosecutions.

Answer (B) is incorrect because numerous companies still engage in anti-competitive conduct.

Answer (C) is incorrect because intent is significant; for example, a company that has a monopoly thrust upon it is not in violation.

Answer (D) is correct. The Sherman Antitrust Act of 1890 makes illegal every contract, combination, or conspiracy in restraint of trade in interstate or foreign commerce. Successful prosecutions have been few because the courts developed a rule of reason stipulating that only unreasonable restraints of trade are illegal. Thus, courts balance the anti-competitive effects against the pro-competitive effects of a restraint of trade. Unless a restraint is unreasonable by its very nature (a per se violation), the rule of reason applies.

[47] Source: CMA 0693 1-28

Answer (A) is incorrect because the ADA bans employment discrimination against people with disabilities.

Answer (B) is incorrect because the ADA provides tax incentives for compliance costs.

Answer (C) is incorrect because the ADA bans discrimination against people with disabilities in transportation.

Answer (D) is correct. Title I prohibits employment discrimination against qualified individuals with disabilities with respect to job applications, hiring, promotion, training, pay, and termination. The Act also establishes requirements for employers and providers of public accommodations, public transportation, and telecommunications. Employers must make "reasonable accommodations" for individuals with disabilities that do not impose an undue burden, for example, by modifying facilities and work schedules, obtaining equipment, etc. However, no federal funds are provided for implementation of ADA by employers.

[48] Source: CMA 0693 1-29

Answer (A) is incorrect because quality standards were established by the Environmental Protection Agency.

Answer (B) is incorrect because fees have been established for firms that emit pollutants.

Answer (C) is correct. Environmental laws have established quality standards with respect to water, land, and air. One approach has permitted violations, but subjects violators to sliding charges depending upon the quantity of pollutants released. Companies that are below the standard for a particular pollutant can sell their rights to other firms in the industry. In addition, tax laws have provided incentives for environmental improvements, for example a tax credit for individuals who installed insulation. Similarly, windmills are encouraged by the tax laws. Moving facilities to offshore sites where governmental standards are less stringent might be undertaken by individual companies, but such action is not a governmental device.

Answer (D) is incorrect because companies may buy and sell their rights to emit pollutants.

[49] Source: CMA 0693 1-30

Answer (A) is incorrect because the FDA can extend patent lives to allow for time lost during the regulatory review.

Answer (B) is incorrect because the FDA considers benefit-risk tradeoffs.

Answer (C) is incorrect because the FDA maintains purity standards.

Answer (D) is correct. The FDA was created by the Federal Food, Drug, and Cosmetics Act of 1938 to

Page 26: CMAPart1D(Domestic Insitutional Environment of Business)

help maintain the safety of drugs, food, cosmetics, and medicinal products and devices. It regulates the testing, distribution, and sale of drugs. Upon receiving a new drug application, the FDA conducts hearings and investigates the merits of the drug in a process that may require years. Thus, an application must be supported by substantial evidence that the drug will have the asserted effects.

[50] Source: CMA 1293 1-1

Answer (A) is incorrect because the Sherman Act also applies to domestic corporations.

Answer (B) is incorrect because the Taft-Hartley Act of 1947 outlawed closed shops.

Answer (C) is incorrect because monopolies in interstate or foreign commerce are illegal; monopolies in intrastate commerce are not covered by the Sherman act. Also, a company must show an objective interest to monopolize. Monopoly power is lawful if it is obtained by superior businesses acumen.

Answer (D) is correct. Section 1 of the Sherman Antitrust Act of 1890 makes illegal every contract, combination, or conspiracy in restraint of trade in interstate or foreign commerce. Some types of arrangements between competitors are found unreasonable without inquiry. These are per se violations and include price fixing, division of markets, group boycotts, and resale price maintenance. Section 2 prohibits the acts of monopolizing or attempting to monopolize.

[51] Source: CMA 1293 1-2

Answer (A) is incorrect because the forces of supply and demand do not determine prices; a regulatory agency sets limits on the price a natural monopoly can charge.

Answer (B) is incorrect because firms cannot easily enter the utility industry. Great amounts of capital are needed to establish a natural monopoly.

Answer (C) is incorrect because quality standards are set by the same regulatory agency that controls prices.

Answer (D) is correct. Regulated industries such as gas and electric utilities, most of which are natural monopolies, have price and quality controls established by state or local governmental regulatory bodies. Such industries usually have inelastic demand curves, which means regulation can help control prices. Public utilities are monopolies because having more than one of each type of utility in a locality would be inefficient. Rates are normally set to provide owners with a reasonable return on their investment.

[52] Source: CMA 1293 1-4

Answer (A) is correct. Social regulation concerns quality of life issues, e.g., workplace and product

safety, environmental degradation, and fair employment practices. The abuses addressed are those that are difficult for market forces to remedy. For example, consumers may purchase products on the basis of price and quality, but without regard to the environmental impact of their production, and unsafe working conditions may be tolerated by individuals who have few opportunities for other employment.

Answer (B) is incorrect because, although consumer and environmental groups may occasionally exercise some lobbying power, they are typically underfunded and would have little impact on legislatures in the absence of an obvious need for social regulation.

Answer (C) is incorrect because there is great difficulty in measuring both the benefits and costs of most social regulation.

Answer (D) is incorrect because no regulation is desirable, but some regulation is necessary when market forces are ineffective.

[53] Source: CMA 1293 1-5

Answer (A) is incorrect because the Clean Air Act does not make pollution illegal; it provides comprehensive regulation of air quality.

Answer (B) is incorrect because regulating the amount of pollution a specific company can emit is not efficient. That is why the emphasis is on the application of air quality standards in the air quality control regions in the states. Moreover, the states are required to identify the major, not all, sources of pollution.

Answer (C) is incorrect because the Act does not promote social awareness about the harm due to pollution; in fact, it allows some pollution if a company is willing to pay to pollute.

Answer (D) is correct. The Clean Air Act allows companies to pollute legally if they have a permit to do so. Firms can buy or sell these permits, and the result is essentially a free market that enables a firm to seek out a technology that is most cost effective. If the price of permits is too high, a firm will buy new equipment that will not pollute. If permits can be acquired at a low cost, a company may decide to continue producing with its old, high-pollution equipment.

[54] Source: CMA 1293 1-6

Answer (A) is incorrect because a merger between firms in different and unrelated markets is a conglomerate merger.

Answer (B) is incorrect because a merger between two or more firms at different stages of the production process is a vertical merger.

Answer (C) is incorrect because a merger between a producer and a supplier is a vertical merger.

Answer (D) is correct. A horizontal merger is one

Page 27: CMAPart1D(Domestic Insitutional Environment of Business)

between competitors in the same market. From the viewpoint of the Justice Department, it is the most closely scrutinized type of merger because it has the greatest tendency to reduce competition.

[55] Source: CMA 1293 1-7

Answer (A) is incorrect because unions raise wages for their members and indirectly reduce wages in the nonunionized labor market.

Answer (B) is correct. Labor unions have typically worked for higher wages and better working conditions for their members. Through collective bargaining and restrictions on membership, unions increase the wage rates of members in comparison with those earned by nonunionized workers. Because of the greater cost of hiring unionized workers, the amount of labor demanded by employers is reduced. The workers that are unemployed as a result may then seek employment in the nonunion labor market, thereby increasing the supply and decreasing the wages in that market.

Answer (C) is incorrect because unions raise wages for their members and indirectly reduce wages in the nonunionized labor market.

Answer (D) is incorrect because unions raise wages for their members and indirectly reduce wages in the nonunionized labor market.

[56] Source: CMA 1293 1-8

Answer (A) is correct. A closed shop is a workplace in which union membership is required as a condition of obtaining employment. The closed shop was outlawed by the Labor-Management Relations Act (Taft-Hartley Act) of 1947. In contrast with a closed shop, a union shop is still allowed in some states. In a union shop, an employee is required to join the union after employment. Still other states have right-to-work laws that allow employees to work at any job without union membership.

Answer (B) is incorrect because, in a closed shop, all employees must be a member of the union.

Answer (C) is incorrect because collective bargaining is necessary in a closed shop. All workers are members of the union.

Answer (D) is incorrect because only workers who are union members have the right to work in a closed shop.

[57] Source: CMA 1293 1-9

Answer (A) is incorrect because a partnership is more complex than a proprietorship, but less difficult to form than a corporation.

Answer (B) is incorrect because a limited partnership can be created only pursuant to a statute. It is a form of business organization unknown at common law. A limited partnership permits investors to avoid personal liability and the duties of day-to-day

management without submitting to the complexities of corporate formation and operation.

Answer (C) is correct. The most common form of business organization is the sole proprietorship. A proprietorship is easily and inexpensively organized because no formalities are required. Moreover, it consists of one person (the proprietor). It is created under common law whenever a person enters into business and does not choose another form or organization.

Answer (D) is incorrect because a corporation is a legal entity with a legal existence separate from its owners; the formation and existence of corporations is entirely regulated by state statute.

[58] Source: CMA 1293 1-11

Answer (A) is incorrect because the elimination of all pollution would probably be too costly for the resulting benefits.

Answer (B) is incorrect because firms are more supportive of cost-benefit regulatory policies.

Answer (C) is incorrect because granting tax relief to those firms which pollute the least is not a cost-benefit policy.

Answer (D) is correct. If environmental policies weigh the costs and benefits of regulation, some pollution will exist because the costs of eliminating all pollution will likely outweigh the benefits. Cost-benefit considerations are reflected in, for example, the establishment of air quality standards that do not attempt to eliminate the incidence of specified pollutants and requirements for control technology that, for the most part, do not mandate use of devices that achieve the maximum pollution reduction. The drawback of a cost-benefit approach is the difficulty of quantifying benefits.

[59] Source: CMA 1293 1-12

Answer (A) is incorrect because the Federal Reserve System regulates banks, but does not insure deposits.

Answer (B) is incorrect because the FSLIC was the federal agency that formerly insured savings and loan associations; the FSLIC went bankrupt as a result of the savings and loan crisis of the 1980s.

Answer (C) is correct. Commercial bank deposits are insured up to $100,000 per account by the Federal Deposit Insurance Corporation (FDIC), a federal agency. It is funded by premiums paid by member banks.

Answer (D) is incorrect because the treasury is not the primary insurer of bank deposits, although it may ultimately be asked by Congress to provide funds if the FDIC encounters financial difficulties.

[60] Source: Publisher

Answer (A) is incorrect because, under NEPA,

Page 28: CMAPart1D(Domestic Insitutional Environment of Business)

federal agencies must give environmental considerations a weight equal to but not greater than that afforded nonenvironmental concerns. NEPA augments the existing powers of federal agencies to deal with these environmental matters.

Answer (B) is correct. The provisions of NEPA focus on federal governmental actions. Federal agencies are specifically directed to incorporate an analysis of environment consequences in their decision-making processes. Actions of private persons are affected by NEPA only when federal involvement (approval, funding, etc.) is necessary before such persons may act (e.g., federal approval before drilling for oil in ocean waters within U.S. jurisdiction). Otherwise, NEPA does not directly concern activities of private persons.

Answer (C) is incorrect because, under NEPA, federal agencies must give environmental considerations a weight equal to but not greater than that afforded nonenvironmental concerns. NEPA augments the existing powers of federal agencies to deal with these environmental matters.

Answer (D) is incorrect because, under NEPA, federal agencies must give environmental considerations a weight equal to but not greater than that afforded nonenvironmental concerns. NEPA augments the existing powers of federal agencies to deal with these environmental matters.

[61] Source: Publisher

Answer (A) is incorrect because the CWA broadly prohibits any discharges of pollutants into waters subject to the jurisdiction of the United States by any person, except in compliance with the Act. Under the CWA, impairment of navigation is irrelevant.

Answer (B) is incorrect because to be subject to federal jurisdiction, and thus the CWA, the waters in which pollutants are discharged must be so-called "navigable waters," which are defined as waters of the United States (including the territorial waters). This broad definition does not encompass all bodies of water located within the bounds of the United States.

Answer (C) is incorrect because the Rivers and Harbors Act of the late 1800s was the first major piece of federal legislation promulgated to protect U.S. waterways. Until the passage of the CWA, the Rivers and Harbors Act was also used to combat pollutive discharges, although its original purpose was to keep waterways clear from obstruct ions to navigation.

Answer (D) is correct. The CWA (1972) substantially amended the Federal Water Pollution Control Act of 1948. It seeks to restore and maintain the physical and biological integrity of the waters of the United States. Its objectives are to render water suitable for recreation and propagation of fish and other wildlife and to eliminate discharges of pollutants.

[62] Source: Publisher

Answer (A) is incorrect because profit percentages are not set by antitrust laws other than to the extent that price discrimination is prohibited.

Answer (B) is incorrect because firms may enter into joint ventures.

Answer (C) is correct. Antitrust laws are designed to promote more efficient allocation of resources, greater choice for consumers, greater business opportunities, fairness in economic behavior, and avoidance of concentrated political power resulting from economic power. Competition results in greater output and lower prices than other market structures.

Answer (D) is incorrect because competition results in greater output and lower prices than other market structures.

[63] Source: Publisher

Answer (A) is incorrect because a pricing agreement among competitors, even if aimed at lowering prices, is illegal per se because it is assumed that such an agreement over the long run will substantially lessen competition.

Answer (B) is incorrect because pricing agreements aimed at eliminating unfair competition and fixing prices are illegal.

Answer (C) is incorrect because it is presumed that in the long run even reasonable and fair agreements will contribute to a lessening of competition.

Answer (D) is correct. No pricing agreements among competitors are legal because the antitrust laws forbid competitors to agree on the price of the products they sell.

[64] Source: Publisher

Answer (A) is incorrect because many cases have held that 60% or less of the market constitutes monopolistic power. Hence, the firm need not hold 100% of the market.

Answer (B) is incorrect because the government need not prove that the defendant controls a particular percentage of the market to establish that an illegal monopoly or attempt to monopolize has occurred.

Answer (C) is correct. The purpose of antitrust laws is to preserve and promote competition. The working definition of monopoly in antitrust law is that the defendant has the power to control prices or exclude competition. Under the Sherman Act, formation of or the attempt to form a monopoly is illegal.

Answer (D) is incorrect because the Sherman Act prohibits monopolies and attempts to monopolize. Thus, proof of contracts, combinations, and conspiracies in restraint of trade is not always necessary.

Page 29: CMAPart1D(Domestic Insitutional Environment of Business)

[65] Source: Publisher

Answer (A) is incorrect because the accounting requirements apply only to publicly held, registered companies under the 1934 act.

Answer (B) is incorrect because the accounting requirements apply only to publicly held, registered companies under the 1934 act.

Answer (C) is correct. The accounting requirements of the FCPA apply to all companies required to register and report under the Securities Exchange Act of 1934. These companies must maintain books, records, and accounts in reasonable detail that accurately and fairly reflect transactions. The FCPA also requires these companies to maintain a system of internal accounting control that provides certain reasonable assurances, including that corporate assets are not used for bribes. If payoffs are made, they must be reflected in the company's records.

Answer (D) is incorrect because the accounting requirements apply only to publicly held, registered companies under the 1934 act.

[66] Source: Publisher

Answer (A) is correct. The main purpose of the Foreign Corrupt Practices Act of 1977 is to prevent

bribery by firms that do business in foreign countries. A major ramification is that it requires all companies that must register with the SEC under the Securities Exchange Act of 1934 to maintain adequate accounting records and a system of internal accounting control.

Answer (B) is incorrect because, although some international accounting standards have been promulgated, they are incomplete and have not gained widespread acceptance.

Answer (C) is incorrect because there are no requirements for providing periodic reports on foreign commerce or foreign political party affiliations.

Answer (D) is incorrect because U.S. auditing standards apply to the conduct of audits by accountants.

[67] Source: Publisher

Answer (A) is incorrect because this is not covered by the provisions in the FCPA.

Answer (B) is incorrect because this is not covered by the provisions in the FCPA.

Answer (C) is incorrect because all U.S. firms are subject to the anti-bribery provisions.

Answer (D) is correct. The Foreign Corrupt Practices Act (FCPA) prohibits any U.S. firm from making bribes to foreign officials to influence official acts. The businesses subject to the FCPA include corporations, partnerships, limited partnerships, business trusts, and unincorporated organizations.

Violations of the FCPA are federal felonies. The penalties are 5 years in prison or a $10,000 fine or both for an officer, director, or shareholder who helps make the bribe.

[68] Source: CIA 0594 IV-67

Answer (A) is incorrect because the easy and inexpensive organization is one of the most appealing characteristics of sole proprietorships.

Answer (B) is incorrect because sole proprietors make all management decisions, and have the rights to all profits as well as the right to sell the business. Thus, the entrepreneur has freedom of action.

Answer (C) is incorrect because, as the sole owner and profit-sharer, a sole proprietor has a strong incentive to manage the business efficiently so as to earn more profits.

Answer (D) is correct. The sole proprietor does not enjoy the benefits of specialization. (S)he must perform all management functions, including all decisions relating to buying and selling, the acquisition and maintenance of personnel, and the technical aspects of production, advertising, and distribution.

[69] Source: CIA 0594 IV-66

Answer (A) is incorrect because a sole proprietorship is owned by one person. The sole proprietor is personally liable for all debts.

Answer (B) is incorrect because general partners share profits and losses of the venture. Debts of a partnership are ultimately the debts of the individual general partners.

Answer (C) is correct. A shareholder owns a property interest in the underlying net assets of the corporation and is entitled to share in its profits. But, unlike a sole proprietor or a general partner, the shareholder is not subject to liability beyond his/her investment.

Answer (D) is incorrect because monopoly is not a legal form of business organization.

[70] Source: CIA 0589 IV-47

Answer (A) is correct. Unlike a corporation, a partnership is not treated as a separate legal entity for purposes of liability. A corporate shareholder is ordinarily liable only to the extent of the investment, but a general partner usually has full personal liability for the obligations of the partnership.

Answer (B) is incorrect because double taxation is a disadvantage of the corporate but not the partnership form of business organization. Corporate income is taxed at the corporate level, and the individual shareholders pay taxes on dividends. A partnership, however, is not a taxable entity.

Answer (C) is incorrect because partnerships can be created without formalities if two or more persons

Page 30: CMAPart1D(Domestic Insitutional Environment of Business)

agree to become associated as co-owners to carry on a business for profit. The agreement need not even be explicit. A corporation, however, must be formed according to specific statutory requirements.

Answer (D) is incorrect because partnerships are not subject to as many regulatory requirements as corporations.

[71] Source: CIA 0593 IV-47

Answer (A) is incorrect because a limited partnership is not used to limit withdrawals.

Answer (B) is incorrect because a limited partnership is not used to limit the duration of the partnership.

Answer (C) is correct. A primary characteristic of limited partnerships is that they must have both general and limited partners. General partners have unlimited liability for the obligations of the partnership and are entitled to manage the business. A limited partner's liability is ordinarily limited to the amount of the capital contribution to the partnership. (S)he does not have a right to participate in management. A limited partner who nevertheless has a role in management may become personally liable for partnership obligations to third parties aware of that role.

Answer (D) is incorrect because a limited partnership is not used to the authority of general partners to manage the business.

[72] Source: CMA 0694 1-16

Answer (A) is incorrect because the Clayton Act of 1914 makes price discrimination, tying contracts, anti-competitive mergers, and interlocking directorates illegal.

Answer (B) is incorrect because unfair competition and deceptive business practices are prohibited by the Federal Trade Commission Act of 1914.

Answer (C) is incorrect because the Clayton Act of 1914 allows the Justice Department to prevent mergers before they occur.

Answer (D) is correct. The Sherman Act makes

illegal every contract, combination, or conspiracy in restraint of trade in interstate or foreign commerce. Section 2 of the act prohibits monopolization or the attempt to monopolize.

[73] Source: CMA 0694 1-17

Answer (A) is incorrect because the closed shop was outlawed by the Taft-Hartley Act.

Answer (B) is incorrect because the Taft-Hartley Act did not require binding arbitration.

Answer (C) is correct. The Labor-Management Relations Act of 1947, better known as the Taft-Hartley Act, placed restraints on unions to even

the balance of power between labor and management. It outlawed the closed shop, which required union membership as a condition of obtaining employment. In addition, the Taft-Hartley Act required unions to bargain in good faith. Also, the President was given authority to obtain an injunction against strikes for an 80-day cooling-off period if national health, welfare, or security was threatened.

Answer (D) is incorrect because the Taft-Hartley Act did not apply to federal employees.

[74] Source: CMA 0694 1-18

Answer (A) is correct. The Robinson-Patman Act prohibits price discrimination with respect to both buyers and sellers. Price discrimination is allowed if it is to meet a competitor's price quote or if justified by the quantity purchased.

Answer (B) is incorrect because cost increases caused by government specification is not an

allowable exception to the Robinson-Patman Act.

Answer (C) is incorrect because the nature of a market is not a factor in price discrimination under the Robinson-Patman Act.

Answer (D) is incorrect because the Justice Department does not give approval for price discrimination.

[75] Source: CMA 1294 1-10

Answer (A) is incorrect because the Sherman Act of 1890 did not mention tying sales or interlocking directorates.

Answer (B) is correct. The Clayton Act of 1914 prohibits mergers or acquisitions of stock that may lessen competition or tend to create a monopoly. Mergers can be prevented by the Justice Department before they occur. Also prohibited by the Clayton Act are sales that prevent the buyer from dealing with the seller's competitors, tying (or tie-in) sales, exclusive dealing, price discrimination among different buyers, and interlocking directorates.

Answer (C) is incorrect because the Antitrust Improvements Act of 1976 amended the antitrust laws in various ways.

Answer (D) is incorrect because the Federal Trade Commission Act of 1914 created the Federal Trade Commission to enforce the Sherman and Clayton acts.

[76] Source: CMA 1294 1-11

Answer (A) is incorrect because the Taft-Hartley Act provides for an 80-day injunction against a work stoppage that might imperil national security.

Answer (B) is correct. The Wagner Act gave employees the right to join unions, bargain collectively with employers, and engage in concerted activities for

Page 31: CMAPart1D(Domestic Insitutional Environment of Business)

collective bargaining. In effect, labor unions were exempted from antitrust laws. The National Labor Relations Board (NLRB) was formed to administer the Act.

Answer (C) is incorrect because the Taft-Hartley Act of 1947 provided for restraints on unions to permit a more even balance of power between labor and management.

Answer (D) is incorrect because the Landrum-Griffin Act of 1959 requires that union officials be elected on a regular basis and that secret ballots be used. It also requires that unions submit financial reports to the government.

[77] Source: CMA 1294 1-12

Answer (A) is incorrect because the Wagner Act of 1935 prohibited employers from interfering with the right of employees to form unions.

Answer (B) is incorrect because the Taft-Hartley Act required unions to bargain in good faith.

Answer (C) is incorrect because the Norris-LaGuardia Act of 1932 outlawed yellow-dog contracts.

Answer (D) is correct. The Taft-Hartley Act placed restraints on unions that resulted in a more even balance of power between labor and management. One provision prohibits unions from coercing employees to become union members. Thus, the closed shop was outlawed. The act also requires unions to bargain in good faith and provides for mediation of disputes. The President of the USA was given authority to obtain an injunction against strikes or lockouts for an 80-day cooling-off period if the national health, welfare, or security is threatened. Also, secondary strikes and featherbedding are prohibited.

[78] Source: CMA 1294 1-13

Answer (A) is incorrect because the FTC was created in 1914 by the Federal Trade Commission Act.

Answer (B) is incorrect because penalties were stated in the 1890 Act.

Answer (C) is incorrect because the Clayton Act of 1914 prohibits mergers that substantially lessen competition.

Answer (D) is correct. The Sherman Act makes illegal every contract, combination, or conspiracy in restraint of trade in interstate or foreign commerce. Some types of arrangements between competitors are found unreasonable without inquiry. These are called per se violations and include price fixing, division of markets, group boycotts, and resale price maintenance.

[79] Source: CMA 1294 1-14

Answer (A) is incorrect because social legislation is criticized when standards are rigid.

Answer (B) is incorrect because compliance costs are typically high.

Answer (C) is incorrect because the free market is normally not relied upon; with certain exceptions, compliance is mandatory.

Answer (D) is correct. Economic regulation usually concerns price and service to the public and is ordinarily industry specific. Social regulation has broader objectives and more pervasive effects. It addresses quality of life issues, such as workplace and product safety, pollution, and fair employment practices, and it applies to most industries. Social regulation has been criticized on the grounds that it is costly, contributes to overregulation, may inhibit innovation, increases inflation and may place a disproportionate burden on small companies, thereby having an anticompetitive effect. Another criticism is that regulators are perceived to have little concern for the relation of marginal benefits and costs. For example, achieving total compliance with the standards for treated wastewater may be twice as costly as attaining 95% of those standards. In other words, the relatively small improvements sometimes sought by regulators may be viewed as excessively costly by those who are regulated.

[80] Source: CMA 1295 1-20

Answer (A) is incorrect because some regulated industries are subject to antitrust laws.

Answer (B) is correct. With respect to antitrust laws, regulated industries are covered unless specifically exempted by statute or the courts. For example, regulated public utilities are specifically exempted because they are viewed as natural monopolies.

Answer (C) is incorrect because the Justice Department must look to statute or the courts to enforce antitrust laws.

Answer (D) is incorrect because some regulated industries, such as utilities, are exempt from antitrust laws.

[81] Source: CMA 1295 1-21

Answer (A) is correct. The purpose of social regulation is to require behavior that is thought to be beneficial for other than purely economic considerations. However, social regulation has costs, which lead to increased prices and inflation, reduced innovation and consumer choice, and decreased productivity. Social regulation is criticized because the marginal benefits may not equal the marginal costs.

Answer (B) is incorrect because enforcement policies are rarely viewed as lenient by those to whom social legislation is directed.

Answer (C) is incorrect because social legislation may show a concern for the quality of life by

Page 32: CMAPart1D(Domestic Insitutional Environment of Business)

regulating the quality of products.

Answer (D) is incorrect because social regulation often involves noncompliance penalties.

[82] Source: CMA 1295 1-22

Answer (A) is incorrect because the act does not apply to export sales.

Answer (B) is incorrect because cost differences related to delivery are a justification for charging different prices.

Answer (C) is incorrect because the act applies only to sales in interstate commerce.

Answer (D) is correct. The Robinson-Patman Act of 1936, an amendment to the Clayton Act, outlaws price discrimination that would lead to restraint of trade. Both buyer and seller can be found guilty of price discrimination under the provisions of the Robinson-Patman Act. Price differentiation between customers is allowed if there is a difference in costs. For instance, quantity discounts are permitted if it can be shown that larger quantities can be shipped with a cost savings. Charging competing wholesalers different prices for similar goods would be a violation of the act.

[83] Source: CMA 1295 1-23

Answer (A) is incorrect because quantity discounts are not prohibited by the Sherman Act.

Answer (B) is incorrect because the Sherman Act does not apply to labor unions.

Answer (C) is incorrect because the Sherman Act does not prohibit mergers; only those that could lead to restraint of trade are outlawed.

Answer (D) is correct. The Sherman Act of 1890 makes illegal every contract, combination, or conspiracy in restraint of trade in interstate or foreign commerce. Some types of arrangements, called per se violations, are considered unreasonable without inquiry. These violations include price fixing, division of markets, group boycotts, and resale price maintenance. Agreeing to submit identical bids on a government contract would be a form of price fixing, and thus a per se violation.

[84] Source: CMA 1295 1-24

Answer (A) is incorrect because this action is prohibited by the Clayton Act.

Answer (B) is incorrect because this action is prohibited by the Clayton Act.

Answer (C) is incorrect because this circumstance is prohibited by the Clayton Act.

Answer (D) is correct. The Clayton Act of 1914 prohibits (1) mergers that may lessen competition or

tend to create a monopoly, (2) sales that prevent the buyer from dealing with the seller's competitors, (3) tie-in sales (requiring a buyer to take other products in order to buy the first product), (4) price discrimination, and (5) interlocking directorates (directors serve on the boards of competing firms). The Clayton Act does not address unfair and deceptive business practices such as false advertising.

[85] Source: CMA 1295 1-25

Answer (A) is correct. The acquisition of a shoe retailer by a shoe manufacturer is an example of vertical integration. Vertical integration is typified by a merger or acquisition involving companies that are in the same industry but at different levels in the supply chain. In other words, one of the companies supplies inputs for the other.

Answer (B) is incorrect because a conglomerate is a company made up of subsidiaries in unrelated industries.

Answer (C) is incorrect because market extension involves expanding into new market areas.

Answer (D) is incorrect because product extension involves selling additional products in the same markets.

[86] Source: CMA 1295 1-26

Answer (A) is incorrect because blue sky laws are state laws.

Answer (B) is incorrect because blue sky laws are state laws.

Answer (C) is correct. Blue sky laws are state laws designed to prevent fraudulent or misleading security issues. The name came from the fact that some of the earliest laws prohibited "everything under the blue skies which is fraudulent."

Answer (D) is incorrect because blue sky laws do not regulate the environment; they regulate sales of investment securities.

[87] Source: CMA 1295 1-27

Answer (A) is incorrect because the Taft-Hartley Act did not address internal affairs such as financial records and reports.

Answer (B) is incorrect because the Wagner Act of 1935 gave more power to unions.

Answer (C) is incorrect because the Securities Exchange Act of 1934 did not address reports by labor unions.

Answer (D) is correct. The Landrum-Griffin Act of 1959 (the Labor Management Reporting and Disclosure Act) requires unions to maintain financial records and submit reports to the federal government. The intent of the act was to extend the

Page 33: CMAPart1D(Domestic Insitutional Environment of Business)

provisions of the National Labor Relations Act to the internal affairs of unions to make the organizations more democratic and give members more rights.

[88] Source: CMA 1295 1-28

Answer (A) is incorrect because labor unions are specifically exempted from antitrust regulation.

Answer (B) is incorrect because intrastate commerce is specifically exempted from antitrust regulation.

Answer (C) is incorrect because patents and copyrights are specifically exempted from antitrust regulation.

Answer (D) is correct. Several types of entities and contracts are exempt from antitrust regulation. These include firms not operating in interstate commerce, labor unions, regulated public utilities, patents and copyrights, agricultural and fishing organizations, financial institutions, transport industries, professional baseball, and companies qualifying under the Export Trading Company Act. The telecommunications industry is not exempt.

[89] Source: CMA 1295 1-29

Answer (A) is incorrect because they are examples of federal agencies enforcing social regulation.

Answer (B) is incorrect because they are examples of federal agencies enforcing social regulation.

Answer (C) is incorrect because they are examples of federal agencies enforcing social regulation.

Answer (D) is correct. Social regulation is intended to benefit society as a whole. The Food and Drug Administration (FDA), National Highway Traffic Safety Administration, Consumer Product Safety Commission, Occupational Safety and Health Administration (OSHA), Environmental Protection Agency (EPA), and Equal Employment Opportunity Commission (EEOC) are all examples of social agencies.

[90] Source: CMA 1295 1-30

Answer (A) is incorrect because the FTC is not concerned with the regulation of foreign trade.

Answer (B) is incorrect because the FTC is not concerned with the regulation of foreign trade.

Answer (C) is correct. The FTC Act of 1914 prohibits unfair methods of competition and unfair or deceptive acts in commerce. The basic objectives are to initiate antitrust actions and protect the consumer public.

Answer (D) is incorrect because price discrimination and unfair trade practices are merely elements of the overall mission of the FTC.

[91] Source: CMA 0685 1-34

Answer (A) is correct. Although unions have probably been as aggressive as ever in recent years, union membership has declined. This decline is attributable to the fact that growth has occurred in the service industries rather than in the manufacturing industries that have been the traditional strongholds of unions.

Answer (B) is incorrect because this is not a compelling reason for the decline in union membership.

Answer (C) is incorrect because this is not a compelling reason for the decline in union membership.

Answer (D) is incorrect because this is not a compelling reason for the decline in union membership.

[92] Source: CMA 0685 1-35

Answer (A) is incorrect because more members of minorities have joined unions as the barriers of discrimination have been removed and as their overall representation in the labor force has increased.

Answer (B) is incorrect because women have joined unions as the barriers of discrimination have been removed and as their overall representation in the labor force has increased.

Answer (C) is correct. Because the growth areas of the economy have been in the service industries, there has been a decline in union membership among blue-collar workers, who are largely in manufacturing and maintenance occupations.

Answer (D) is incorrect because public sector employment continues to grow while manufacturing sector employment declines.

[93] Source: Publisher

Answer (A) is incorrect because a perfectly competitive market was envisioned by classical economics. Modern monopolies and oligopolies often must be subject to regulation to enforce their social responsibilities since the limits imposed by the market are ineffective.

Answer (B) is incorrect because the concept embraces the public or societal interest.

Answer (C) is correct. The concept of corporate social responsibility involves more than serving the interests of the organization and its shareholders. Rather, it is an extension of responsibility to embrace service to the public interest in such matters as environmental protection, employee safety, civil rights, and community involvement.

Answer (D) is incorrect because the concept embraces the public or societal interest.

Page 34: CMAPart1D(Domestic Insitutional Environment of Business)

[94] Source: Publisher

Answer (A) is incorrect because such behavior may prevent governmental action.

Answer (B) is incorrect because it is an argument for such behavior.

Answer (C) is incorrect because it is an argument for such behavior.

Answer (D) is correct. Socially responsible behavior clearly has immediate costs to the entity, for example, the expenses incurred in affirmative action programs, pollution control, and improvements in worker safety. When one firm incurs such costs and its competitor does not, the other may be able to sell its products or services more cheaply and increase its market share at the expense of the socially responsible firm. The rebuttal argument is that in the long run the socially responsible company may maximize profits by creating goodwill and avoiding or anticipating governmental regulation.

[95] Source: CMA 0696 1-18

Answer (A) is incorrect because horizontal price fixing occurs between two competitors at the same level in the distribution process (e.g., between two retailers in the same industry).

Answer (B) is correct. Resale price maintenance agreements are a form of vertical price fixing because a manufacturer or wholesaler restricts the price that may be charged by a retailer. Such agreements are per se violations of the Sherman Act of 1890.

Answer (C) is incorrect because preemptive buying is not associated with resale price maintenance.

Answer (D) is incorrect because tying arrangements require a buyer to purchase other products in addition to the one that is desired.

[96] Source: CMA 0696 1-21

Answer (A) is incorrect because the SEC does not have to approve a trust indenture.

Answer (B) is incorrect because the 1933 act requires disclosure of nonexempted new issuances of securities, including those of public utility holding companies, not registration of particular entities.

Answer (C) is correct. The Securities Act of 1933 was designed to provide complete and fair disclosure to potential investors. The 1933 act applies only to the initial issuance of securities. Disclosure is accomplished through the requirement that a registration statement be filed with the SEC. Once potential investors have complete disclosure, the assumption is that they can make a reasonable decision.

Answer (D) is incorrect because the Securities Exchange Act of 1934 requires registration of brokers.

[97] Source: CMA 0696 1-22

Answer (A) is incorrect because the closed shop was prohibited by the Taft-Hartley Act of 1947.

Answer (B) is correct. The Clayton Act of 1914 prohibits price discrimination if it directly or indirectly lessens competition such that it tends to create a monopoly. The Robinson-Patman Act amended the Clayton Act to prohibit sellers of goods from granting, and buyers from inducing, unfair discounts and other preferences. However, price differentials are justified by a cost savings to the seller or a good faith effort to meet, but not undercut, the lawful price of a competitor.

Answer (C) is incorrect because group boycotts were prohibited by the Sherman Act of 1890.

Answer (D) is incorrect because oligopolies are not prohibited by the Clayton Act.

[98] Source: CMA 0696 1-23

Answer (A) is incorrect because interlocking directorates are prohibited by the Clayton Act.

Answer (B) is incorrect because price discrimination is prohibited by the Clayton Act as amended by the Robinson-Patman Act.

Answer (C) is incorrect because tying contracts are prohibited by the Clayton Act.

Answer (D) is correct. The Federal Trade Commission Act of 1914, as amended by the Wheeler-Lea Amendment of 1938, prohibits unfair methods of competition and unfair or deceptive acts in commerce, including false advertising. The act also created the Federal Trade Commission (FTC) to enforce the requirements of the act. The basic objectives of the FTC are to initiate antitrust actions and to protect the consumer public.

[99] Source: CMA 0696 1-24

Answer (A) is correct. The CPSC publishes product safety standards, issues rules banning certain hazardous products, brings federal suits to eliminate dangers presented by imminently hazardous consumer products, and requires manufacturers, distributors, and retailers to give notice if they have reason to know that their products present a substantial hazard. Remedies include injunctive relief, seizure of products, and civil and criminal penalties.

Answer (B) is incorrect because the CPSC must file suit in federal court to have a product banned or seized; it cannot ban or seize a product without a court order.

Answer (C) is incorrect because the CPSC must file suit in federal court to have a product banned or seized; it cannot ban or seize a product without a court order.

Page 35: CMAPart1D(Domestic Insitutional Environment of Business)

Answer (D) is incorrect because the CPSC may go beyond asking for voluntary action.

[100] Source: CMA 0696 1-25

Answer (A) is incorrect because age may not be asked under the provisions of the Age Discrimination in Employment Act.

Answer (B) is incorrect because questions about a disability may not be asked under the provisions of the Americans with Disabilities Act of 1990.

Answer (C) is incorrect because marital status is not a suitable subject for questioning under the civil rights laws.

Answer (D) is correct. The Equal Employment Opportunity Commission (EEOC) specifies several questions that cannot be asked of applicants during an employment interview. These include questions regarding age, disability, arrest record, and marital status. It is permissible to ask an applicant for references.

[101] Source: CMA 0696 1-26

Answer (A) is incorrect because a closed shop requires an employee to be a member of the union in order to get a job.

Answer (B) is incorrect because a union shop requires an employee to join the union after employment.

Answer (C) is correct. Some states have right-to-work laws that allow employees to work at any job without union membership. Thus, employees can decide not to be involved with a union even though the union represents the workers in the shop.

Answer (D) is incorrect because paying dues is equivalent to the requirements of a union shop.

[102] Source: CMA 0696 1-27

Answer (A) is incorrect because no labeling requirements exist in an unregulated market.

Answer (B) is correct. Mandatory labeling follows the disclosure approach. The basis of this approach is the belief that regulation is sufficient if consumers have the information needed to make rational decisions. The assumptions are that consumers are sophisticated enough to use the information provided and that sellers will have an incentive to improve their products.

Answer (C) is incorrect because stricter liability rules promote product quality and safety, whereas labeling requirements are primarily informational.

Answer (D) is incorrect because labeling requirements do not create marketable property rights. An example of such a regulatory practice is the external trading of pollution rights.

[103] Source: CMA 0696 1-28

Answer (A) is incorrect because constitutional law is

the fundamental law of a jurisdiction.

Answer (B) is incorrect because statutory law is a body of detailed enactments by the legislative branch.

Answer (C) is correct. Administrative law is promulgated by the executive branch under a general grant of authority to an agency to regulate an industry. Administrative law may also be promulgated under a specific grant of authority to an agency to make detailed rules to achieve the objectives of a statute. For example, the IRS makes rules to carry out specific statutes but is not given the general authority to make rules for the collection of revenue. Administrative law may not go beyond the scope of the statutes under which it is promulgated.

Answer (D) is incorrect because judicial (common) law is created by the courts through the adjudication of cases and the publication of the resulting opinions.

[104] Source: CMA 1296 1-20

Answer (A) is incorrect because the

Robinson-Patman Act concerns price discrimination, not exclusive-dealing contracts.

Answer (B) is incorrect because exclusive-dealing contracts are the subject of federal antitrust law. They are not illegal under common law.

Answer (C) is incorrect because there is no requirement to seek approval from the FTC.

Answer (D) is correct. The Clayton Act of 1914 prohibits exclusive-dealing requirements. However, a franchise relationship is contractual, and the agreement is in force for a specified period. Because the franchisee and the franchisor have a common public identity, an exclusive-dealing contract is allowable if it is necessary to assure product quality. Without the exclusive-dealing requirement, a low-quality product sold by the franchisee could reflect badly on the reputation of the franchisor.

[105] Source: CMA 1296 1-21

Answer (A) is correct. The basic purpose of the federal securities laws in the United States, primarily the Securities Act of 1933 and the Securities Exchange Act of 1934, is to provide complete and fair disclosure to potential investors. The emphasis is on disclosure that allows informed investors to make intelligent decisions.

Answer (B) is incorrect because the SEC does not evaluate the merits of investments or define an investment-grade security.

Answer (C) is incorrect because a corporation's by-laws provide for voting rights. For example, preferred stock rarely has a vote.

Page 36: CMAPart1D(Domestic Insitutional Environment of Business)

Answer (D) is incorrect because the federal laws apply in all states regardless of the existence of applicable state laws.

[106] Source: CMA 1296 1-22

Answer (A) is incorrect because the FTC was established by the Federal Trade Commission Act of 1914.

Answer (B) is correct. The Sherman Act of 1890 makes illegal every contract, combination, or conspiracy in restraint of trade in interstate or foreign commerce. Some types of arrangements between competitors are to be considered unreasonable without inquiry. These are known as per se violations. Price fixing, division of markets, group boycotts, and resale price maintenance are per se violations.

Answer (C) is incorrect because price discrimination is prohibited by the Robinson-Patman Act of 1936, which was an amendment to the Clayton Act of 1914.

Answer (D) is incorrect because patent laws were in effect long before the Sherman Act of 1890. Thus, the U.S. Constitution (Article I, Section 8) grants to Congress the power "To promote the progress of science and useful arts, by securing for limited times to authors and inventors the exclusive right to their respective writings and discoveries."

[107] Source: CMA 1296 1-23

Answer (A) is incorrect because the Clayton Act of 1914 prohibits interlocking directorates in competing companies.

Answer (B) is incorrect because the Robinson-Patman Act of 1936 prohibits price discrimination.

Answer (C) is correct. The Antitrust Improvements Act of 1976 requires corporations with annual sales or assets exceeding $100,000,000 to give advance notice to the Justice Department and the FTC of any acquisition of a corporation with annual sales or assets of $10,000,000 or more.

Answer (D) is incorrect because the Sherman Act of 1890 prohibits restraint of trade and monopoly.

[108] Source: CMA 1296 1-24

Answer (A) is incorrect because the Age Discrimination in Employment Act provides protection for older workers.

Answer (B) is incorrect because sunset laws are not concerned with securities regulation.

Answer (C) is incorrect because laws that prohibit the sale of alcohol on Sundays are blue laws.

Answer (D) is correct. Sunset laws are provisions that require periodic review and reenactment of

specific laws. Otherwise, the legislation or agency terminates at the end of a specified period of time. For example, the first federal Cost Accounting Standards Board was established with a 10-year life. At the end of that period, the agency was terminated because Congress felt that its purpose had been accomplished.

[109] Source: CMA 1296 1-25

Answer (A) is incorrect because shareholders in publicly traded U. S. corporations have the right to vote on fundamental corporate changes, to receive declared dividends and annual reports, to vote, to exercise any preemptive right that may have been granted, to attend meetings, to inspect corporate records, and to bring shareholder suits.

Answer (B) is incorrect because shareholders in publicly traded U. S. corporations have the right to vote on fundamental corporate changes, to receive declared dividends and annual reports, to vote, to exercise any preemptive right that may have been granted, to attend meetings, to inspect corporate records, and to bring shareholder suits.

Answer (C) is incorrect because shareholders in publicly traded U. S. corporations have the right to vote on fundamental corporate changes, to receive declared dividends and annual reports, to vote, to exercise any preemptive right that may have been granted, to attend meetings, to inspect corporate records, and to bring shareholder suits.

Answer (D) is correct. A corporation is owned by shareholders who elect a board of directors to manage the company. The board of directors then hires managers to supervise operations. Shareholders do not vote on major management changes because the powers of the board include selection and removal of officers and the setting of management compensation. Shareholders do have the right to vote on fundamental corporate changes, e.g., mergers and acquisitions, any changes in the corporate charter and bylaws, and dissolution.

[110] Source: CMA 1296 1-29

Answer (A) is incorrect because the ADA prohibits employers from inquiring about a job applicant's prior health insurance claims.

Answer (B) is incorrect because the ADA applies to both physical and mental impairments.

Answer (C) is correct. The ADA requires organizations with 25 or more employees to provide reasonable accommodation for employees and job applicants with disabilities. The ADA bans employment discrimination against people with mental or physical disabilities, provides tax incentives for compliance costs, and requires remodeling of facilities to provide access by individuals with disabilities.

Another provision of the ADA is that employers are prohibited from inquiring into a job applicant's disability with questions concerning medical history, prior workers' compensation or health insurance

Page 37: CMAPart1D(Domestic Insitutional Environment of Business)

claims, work absenteeism due to illness, past treatment for alcoholism, or mental illness.

Answer (D) is incorrect because the ADA provides no federal funds for implementation of its provisions.

[111] Source: CMA 1296 1-30

Answer (A) is incorrect because monetary penalties were imposed for insider trading before 1988. For example, the Securities Exchange Act of 1934 requires insiders to turn over to the corporation any short-swing profits on purchases and sales of corporate stock.

Answer (B) is correct. The Insider Trading Sanctions Act of 1984 extended the law to include those who aid and abet insider trading. It also imposed a treble-damages penalty. The 1988 act increased prison terms from 5 to 10 years, the maximum fine for individuals to $1,000,000, and the maximum fine for corporations to $2,500,000. In addition, the SEC now has authority to reward an informant with up to 10% of the fine levied against the perpetrator.

Answer (C) is incorrect because, under section 10(b) of the 1934 act and Rule 10b-5, insider trading is a fraudulent act subject to criminal penalties.

Answer (D) is incorrect because the SEC already held the right to recover gains resulting from the illegal use of insider information.

[112] Source: CMA 0697 1-22

Answer (A) is correct. SEC Rule 415 allows corporations to file registration statements covering a stipulated amount of securities that may be issued over the 2-year effective period of the statement. The securities are placed on the shelf and issued at an opportune moment without the necessity of filing a new registration statement, observing a 20-day waiting period, or preparing a new prospectus. The issuer is only required to provide updating amendments or to refer investors to quarterly and annual statements filed with the SEC. Shelf registration is most advantageous to large corporations that frequently offer securities to the public.

Answer (B) is incorrect because an indenture is a contract associated with the issuance of a debt instrument.

Answer (C) is incorrect because a secondary market registration does not expedite the issuance of securities.

Answer (D) is incorrect because a red-herring is a preliminary prospectus issued during the 20-day waiting period before the related registration statement becomes effective.

[113] Source: CMA 0697 1-25

Answer (A) is incorrect because the Celler-Kefauver

Act of 1950 prohibits the acquisition of the stock or assets of another business if the effect may be to lessen competition substantially or to create a monopoly.

Answer (B) is incorrect because the Federal Trade Commission Act of 1914 prohibits unfair methods of competition in or affecting interstate commerce. It also created the Federal Trade Commission, which has authority to enforce the Clayton and Robinson-Patman Acts. The FTC also is authorized to proceed against unfair or deceptive acts or practices.

Answer (C) is incorrect because the Sherman Act of 1890 makes illegal every contract, combination, or conspiracy that unreasonably restrains trade in interstate or foreign commerce. It also prohibits monopolization and attempts and conspiracies to monopolize.

Answer (D) is correct. The Robinson-Patman Act of 1936 amended the Clayton Act with respect to price discrimination. Price discrimination by both buyers and sellers is prohibited in interstate commerce of goods of like grade and quality. The purpose of the act is to protect competition. However, price differentials are allowed if justified by a cost savings to the seller or a good-faith effort to meet a competitor's lawful price.

[114] Source: CMA 0697 1-26

Answer (A) is incorrect because the Federal Trade Commission Act of 1914 prohibits unfair methods of competition in or affecting interstate commerce. It also created the Federal Trade Commission, which has authority to enforce the Clayton and Robinson-Patman Acts. The FTC also is authorized to proceed against unfair or deceptive acts or practices.

Answer (B) is correct. The Securities Exchange Act of 1934 addresses the issue of insider trading. Specifically, insiders must turn over to the corporation any profits earned on purchases and sales of their company's stock that fall within six months of each other. They are also prohibited from buying or selling stock based on inside information not available to the public.

Answer (C) is incorrect because the Clayton Act of 1914 was intended to prevent monopolies. A probability of a significant anticompetitive effect is a basis for most violations of the act. It specifically addresses price discrimination, tying contracts, exclusive dealing arrangements, mergers, and interlocking directorates.

Answer (D) is incorrect because the North American Free Trade Agreement is an agreement providing for free trade among the USA, Canada, and Mexico.

[115] Source: CMA 0697 1-27

Answer (A) is incorrect because on-time departure standards are an aspect of the airline industry that has

Page 38: CMAPart1D(Domestic Insitutional Environment of Business)

been deregulated.

Answer (B) is incorrect because domestic fare schedules are an aspect of the airline industry that has been deregulated.

Answer (C) is correct. Airline operations have been almost fully deregulated by the U. S. government, for example, with regard to departure standards, fare schedules, and the opportunity to eliminate

unprofitable routes. Despite deregulation, however, airlines are still subject to federal and state labor laws, including those governing collective bargaining. The National Labor Relations Board exercises its jurisdiction over all transportation enterprises that furnish interstate services.

Answer (D) is incorrect because international fare schedules are an aspect of the airline industry that has been deregulated.

[116] Source: CMA 0697 1-28

Answer (A) is correct. Federal administrative agencies belong to the executive branch of government. They have quasi-legislative powers (rulemaking authority pursuant to enabling statutes enacted by Congress), quasi-judicial powers (the ability to adjudicate certain disputes within their jurisdiction), and quasi-executive powers (for example, to investigate and prosecute violations). However, no federal agency has taxing authority, a power reserved to Congress. Moreover, all bills for raising revenue must originate in the House of Representatives.

Answer (B) is incorrect because it is a frequently heard criticism of federal regulatory agencies and policies.

Answer (C) is incorrect because it is a frequently heard criticism of federal regulatory agencies and policies.

Answer (D) is incorrect because it is a frequently heard criticism of federal regulatory agencies and policies.

[117] Source: CMA 0697 1-29

Answer (A) is incorrect because insider trading is prohibited by Section 10(b) of the Securities Exchange Act of 1934 and by the SEC's Rule 10b-5.

Answer (B) is incorrect because an issuer that wishes to make an interstate offering of new securities to the public must file a registration statement with the SEC.

Answer (C) is incorrect because the Securities Exchange Act of 1934 provides rules with regard to proxy solicitations and tender offers.

Answer (D) is correct. The SEC is charged with enforcement of federal securities laws. Under the securities Act of 1933, the offer or sale of a security to the public requires registration with the SEC absent a specific exemption. However, the 1933 act

is essentially a disclosure statute. The SEC does not evaluate the merits of securities. Its role is to enforce the laws ensuring the public availability of information to potential investors.

[118] Source: Publisher

Answer (A) is incorrect because the definition of the term "hazardous substance" as used in CERCLA does not include petroleum or any derivatives thereof or natural gas.

Answer (B) is incorrect because the definition of the term "hazardous substance" as used in CERCLA does not include petroleum or any derivatives thereof or natural gas.

Answer (C) is correct. Asbestos is a hazardous substance subject to the provisions of CERCLA.

Answer (D) is incorrect because the definition of the term "hazardous substance" as used in CERCLA does not include petroleum or any derivatives thereof or natural gas.

[119] Source: Publisher

Answer (A) is incorrect because under NEPA, federal agencies must give environmental considerations a weight equal to, but not greater than, that afforded nonenvironmental concerns.

Answer (B) is correct. The provisions of NEPA focus on federal governmental actions. Federal agencies are specifically directed to incorporate an analysis of environmental consequences in their decision-making processes. Actions of private persons are affected by NEPA only when federal involvement (approval, funding, etc.) is necessary before such persons may act (e.g., federal approval before drilling for oil in ocean waters within U.S. jurisdiction). Otherwise, NEPA does not directly concern activities of private persons.

Answer (C) is incorrect because under NEPA, federal agencies must give environmental considerations a weight equal to, but not greater than, that afforded nonenvironmental concerns.

Answer (D) is incorrect because NEPA augments the existing powers of federal agencies to deal with these environmental matters.

[120] Source: Publisher

Answer (A) is incorrect because the CWA broadly prohibits any discharges of pollutants into waters, except if in compliance with the act. Impairment of navigation is irrelevant.

Answer (B) is incorrect because, to be subject to the CWA, the waters must be "navigable waters."

Answer (C) is incorrect because the Rivers and Harbors Act of the late 1800s was used to combat pollutive discharges, although its original purpose was to keep waterways clear from obstructions to

Page 39: CMAPart1D(Domestic Insitutional Environment of Business)

navigation.

Answer (D) is correct. The CWA (1972) substantially amended the Federal Water Pollution Control Act of 1948. It seeks to restore and maintain the physical and biological integrity of the waters of the United States. Its objectives are to render water suitable for recreation and propagation of fish and other wildlife and to eliminate discharges of pollutants.

[121] Source: CMA 1294 1-9

Answer (A) is incorrect because no information is given regarding costs. Only selling prices and quantities are given.

Answer (B) is incorrect because AC represents greater satisfied demand.

Answer (C) is correct. The area ABC represents the efficiency gain from antitrust action. It is the increase in productivity (benefits to society) from the increased quantity and lower prices resulting from the antitrust action.

Answer (D) is incorrect because total benefit is represented by the greater quantity produced (area CBQ2Q1).

[122] Source: Publisher

Answer (A) is incorrect because "practitioners of management accounting and financial management have an obligation to the public, their profession, the organization they serve, and themselves, to maintain the highest standards of ethical conduct."

Answer (B) is incorrect because the audit committee would be consulted first only if it were the next higher managerial level.

Answer (C) is correct. To resolve an ethical problem, the financial manager/management accountant's first step is usually to consult his/her immediate superior. If that individual is involved, the matter should be taken to the next higher level of management.

Answer (D) is incorrect because if the superior is involved, the next higher managerial level should be consulted first.

[123] Source: Publisher

Answer (A) is incorrect because a perfectly competitive market was envisioned by classical economics.

Answer (B) is incorrect because the concept embraces the public or societal interest.

Answer (C) is correct. The concept of corporate social responsibility involves more than serving the interests of the organization and its shareholders. Rather, it is an extension of responsibility to embrace service to the public interest in such matters as environmental protection, employee safety, civil

rights, and community involvement.

Answer (D) is incorrect because the concept embraces the public or societal interest.

[124] Source: Publisher

Answer (A) is incorrect because such behavior may prevent governmental action.

Answer (B) is incorrect because each is an argument for such behavior.

Answer (C) is incorrect because each is an argument for such behavior.

Answer (D) is correct. Socially responsible behavior clearly has immediate costs to the entity, for example, the expenses incurred in affirmative action programs, pollution control, and improvements in worker safety. When one firm incurs such costs and its competitor does not, the other may be able to sell its products or services more cheaply and increase its market share at the expense of the socially responsible firm. The rebuttal argument is that in the long run the socially responsible company may maximize profits by creating goodwill and avoiding or anticipating governmental regulation.

[125] Source: Publisher

Answer (A) is incorrect because it states an aspect of the competence requirement.

Answer (B) is correct. According to the IMA Code of Ethics, financial managers/management accountants must "avoid actual or apparent conflicts of interest and advise all appropriate parties of any potential conflict."

Answer (C) is incorrect because it states an aspect of the confidentiality requirement.

Answer (D) is incorrect because it states an aspect of the competence requirement.

[126] Source: Publisher

Answer (A) is incorrect because the code does not address these matters.

Answer (B) is incorrect because the code does not address these matters.

Answer (C) is correct. Financial managers/management accountants may not dis close confidential information acquired in the course of their work unless authorized or legally obligated to do so. They must inform subordinates about the confidentiality of information and monitor their activities to maintain that confidentiality. Moreover, financial managers/management accountants should avoid even the appearance of using confidential information to their unethical or illegal advantage.

Answer (D) is incorrect because other employment may be accepted unless it constitutes a conflict of

Page 40: CMAPart1D(Domestic Insitutional Environment of Business)

interest.

[127] Source: CMA 1

Answer (A) is incorrect because the competence standard pertains to the financial manager/management accountant's responsibility to maintain his/her professional skills and knowledge. It also pertains to the performance of activities in a professional manner.

Answer (B) is incorrect because the confidentiality standard concerns the financial manager/management accountant's responsibility not to disclose or use the firm's confidential information.

Answer (C) is correct. One of the responsibilities of the financial manager/management accountant under the integrity standard is to "recognize and communicate professional limitations or other constraints that would preclude responsible judgment or successful performance of an activity."

Answer (D) is incorrect because objectivity is the fourth part of the IMA Code of Ethics. It requires that information be communicated "fairly and objectively," and that all information that could reasonably influence users be fully disclosed.

[128] Source: CMA 2

Answer (A) is incorrect because the competence standard pertains to the financial manager/management accountant's responsibility to maintain his/her professional skills and knowledge. It also pertains to the performance of activities in a professional manner.

Answer (B) is incorrect because the confidentiality standard concerns the financial manager/management accountant's responsibility not to disclose or use the firm's confidential information.

Answer (C) is correct. The integrity standard requires the financial manager/management accountant to "refuse any gift, favor, or hospitality that would influence or would appear to influence his/her actions.

Answer (D) is incorrect because objectivity is the fourth part of the IMA Code of Ethics. It requires that information be communicated "fairly and objectively," and that all information that could reasonably influence users be fully disclosed.